Test 3

¡Supera tus tareas y exámenes ahora con Quizwiz!

A highly agitated client paces the unit and states, "I could buy and sell this place." The client's mood fluctuates from fits of laughter to outbursts of anger. Which is the most accurate documentation of this client's behavior?

"Agitated and pacing. Exhibiting grandiosity. Mood labile."

A highly agitated client paces the unit and states, "I could buy and sell this place." The client's mood fluctuates from fits of laughter to outbursts of anger. Which is the most accurate documentation of this client's behavior? A. "Rates mood 8/10. Exhibiting looseness of association. Euphoric." B. "Mood euthymic. Exhibiting magical thinking. Restless." C. "Mood labile. Exhibiting delusions of reference. Hyperactive." D. "Agitated and pacing. Exhibiting grandiosity. Mood labile."

"Agitated and pacing. Exhibiting grandiosity. Mood labile."

What tool should a nurse use to differentiate occasional spontaneous behaviors of children from behaviors associated with bipolar disorder?

"FIND" tool

A client's spouse asks, "What evidence supports the possibility of genetic transmission of bipolar disorder?" Which is the best nursing reply? A. "Clients diagnosed with Bipolar Disorders have alterations in neurochemicals that affect behaviors." B. "Higher rates of relatives diagnosed with Bipolar Disorder are found in families of clients diagnosed with this disorder." C. "Higher rates of relatives of clients diagnosed with Bipolar Disorder respond in an exaggerated way to daily stress." D. "More individuals diagnosed with Bipolar Disorder come from higher socioeconomic and educational backgrounds."

"Higher rates of relatives diagnosed with Bipolar Disorder are found in families of clients diagnosed with this disorder."

A client's spouse asks, "What evidence supports the possibility of genetic transmission of bipolar disorder?" Which is the best nursing reply?

"Higher rates of relatives diagnosed with bipolar disorder are found in families of clients diagnosed with this disorder."

Which client statement indicates to the nurse that the client understands dietary teaching related to lithium carbonate (Lithobid) treatment? A. "I will limit my intake of fluids daily." B. "I will maintain normal salt intake." C. "I will take Lithobid on an empty stomach." D. "I will increase my caloric intake to prevent weight loss."

"I will maintain normal salt intake."

Which client statement would the nurse recognize as indicating that the client understands dietary teaching related to lithium carbonate (Lithobid) treatment?

"I will maintain normal salt intake."

The mental health nurse is providing discharge teaching for a client diagnosed with Bipolar Disorder. Which statement indicates that the nurse's teaching is effective? A. "I shouldn't take my lithium when I have the flu." B. "I am looking forward to having real coffee in the morning." C. "I can get off medication in 5 years if I am stable." D. "I'll be the designated driver since I shouldn't have alcohol with lamotrigine."

"I'll be the designated driver since I shouldn't have alcohol with lamotrigine."

After teaching a client about lithium carbonate (Lithane), a nurse would consider the teaching successful on the basis of which client statement?

"I'll call my doctor immediately if I experience any diarrhea or ringing in my ears."

After teaching a client about lithium carbonate (Lithane), the nurse would conclude teaching was successful based on which client statement? A. "I should expect to feel better in a couple of days." B. "I'll call my doctor immediately if I experience any diarrhea or ringing in my ears." C. "If I forget a dose, I can double the dose the next time I take this drug." D. "I need to restrict my intake of any food containing salt."

"I'll call my doctor immediately if I experience any diarrhea or ringing in my ears."

A nurse learns at report that a newly admitted client experiencing mania is demonstrating grandiose delusions. The nurse should recognize that which client statement would provide supportive evidence of this symptom?

"I'm the world's most perceptive attorney."

The nurse learns at report that a newly admitted client experiencing mania is demonstrating grandiose delusions. The nurse should recognize that which client statement provides supportive evidence of this symptom? A. "I can't stop my sexual urges. They have led me to numerous affairs." B. "I'm the world's most perceptive attorney." C. "My wife is distraught about my overspending." D. "The FBI is out to get me."

"I'm the world's most perceptive attorney."

A nursing instructor is teaching about the prevalence of bipolar disorder. Which student statement indicates learning has occurred? A. "This disorder is more prevalent in lower socioeconomic groups." B. "This disorder is more prevalent in higher socioeconomic groups." C. "This disorder is equally prevalent in all socioeconomic groups." D. "This disorder's prevalence cannot be evaluated on the basis of socioeconomic groups."

"This disorder is more prevalent in higher socioeconomic groups."

A nursing instructor is teaching about the prevalence of bipolar disorder. Which student statement indicates that learning has occurred?

"This disorder is more prevalent in the higher socioeconomic groups."

An adult client diagnosed with Bipolar I Disorder is prescribed lamotrigine (Lamictal), 400 mg three times a day, for mood stabilization. Which statement about this medication order is true? A. "This dosage is within the recommended dosage range." B. "This dosage is lower than the recommended dosage range." C. "This dosage is more than twice the recommended dosage range." D. "This dosage is four times higher than the recommended dosage range."

"This dosage is more than twice the recommended dosage range."

A nursing instructor is discussing various challenges in the treatment of clients diagnosed with Bipolar Disorder. Which student statement demonstrates an understanding of the most critical challenge in the care of these clients? A. "Treatment is compromised when clients can't sleep." B. "Treatment is compromised when irritability interferes with social interactions." C. "Treatment is compromised when clients have no insight into their problems." D. "Treatment is compromised when clients choose not to take their medications."

"Treatment is compromised when clients choose not to take their medications."

A nursing instructor is discussing various challenges in the treatment of clients diagnosed with bipolar disorder. Which student statement demonstrates an understanding of the most critical challenge in the care of these clients?

"Treatment is compromised when clients choose not to take their medications."

A client began taking lithium for the treatment of bipolar disorder approximately 1 month ago. The client asks if it is normal to have gained 12 pounds in this time frame. Which is the appropriate nursing reply?

"Weight gain is a common but troubling side effect."

A client began taking lithium for the treatment of bipolar disorder approximately 1 month ago. The client asks if it is normal to have gained 12 pounds in this time frame. Which is the appropriate nursing reply? A. "That's strange. Weight loss is the typical pattern." B. "What have you been eating? Weight gain is not usually associated with lithium." C. "Weight gain is a common but troubling side effect." D. "Weight gain occurs only during the first month of treatment with this drug."

"Weight gain is a common but troubling side effect."

A client diagnosed with Bipolar I Disorder is exhibiting severe manic behaviors. A physician prescribes lithium carbonate (Eskalith) and olanzapine (Zyprexa). The client's spouse questions the Zyprexa order. Which is the appropriate nursing reply? A. "Zyprexa in combination with Eskalith cures manic symptoms." B. "Zyprexa prevents extrapyramidal side effects." C. "Zyprexa ensures a good night's sleep." D. "Zyprexa calms hyperactivity until the Eskalith takes effect."

"Zyprexa calms hyperactivity until the Eskalith takes effect."

A client diagnosed with bipolar I disorder is exhibiting severe manic behaviors. A physician prescribes lithium carbonate (Eskalith) and olanzapine (Zyprexa). The client's spouse questions the Zyprexa order. Which is the appropriate nursing reply?

"Zyprexa calms hyperactivity until the Eskalith takes effect."

A nurse begins the intake assessment of a client diagnosed with bipolar I disorder. The client shouts, "You can't do this to me. Do you know who I am?" Which is the priority nursing action in this situation?

To provide self and client with a safe environment

What is a risk factor for developing​ obsessive-compulsive disorder​ (OCD)? a Vaccinations b Sexual abuse c Allergies d Substance abuse

b (One of the risk factors for developing OCD is childhood sexual abuse.​ Vaccinations, allergies, and substance abuse are not risk factors for developing OCD.)

62. Cliff has been attending group counseling for depression and has been expressing more hopelessness in the last few days. When the nurse provides the group with a homework assignment to be completed and returned to the group the next day, Cliff responds "I don't need to bother." Which of these responses by the nurse is most appropriate? 1) "Are you having suicidal thoughts?" 2) "Trust me, it will be beneficial." 3) "Why don't you want to cooperate?" 4) "This assignment will help you combat the hopelessness."

1) "Are you having suicidal thoughts?" Hopelessness is a risk factor for suicide, and the client's statement may be a veiled suicide threat, so it is most important to assess for suicide risk in response.

56. A client being treated for depression asks the nurse what causes this illness. Which response by the nurse is the most accurate, evidence-based statement? 1) "The etiology of depression is unclear. Evidence supports there may be several different causative factors." 2) "Depression has been proven to be the result of an imbalance in certain neurotransmitters." 3) "Depression is transmitted by a specific gene for the illness." 4) "Depression has been proven to develop as a result of negative thinking patterns."

1) "The etiology of depression is unclear. Evidence supports there may be several different causative factors." Although several theories have been advanced, no single cause for depression has been identified conclusively.

61. Bill is a 70-year-old man who is diagnosed with major depressive disorder. He is married and has two adult children who are alcoholics. He currently lives in a rural neighborhood and works part-time at a convenience mart. Which of these demographics is a risk factor for suicide? 1) 70-year-old male 2) Parent of alcoholic children 3) Lives in a rural neighborhood 4) Works part-time

1) 70-year-old male Suicide is highest among persons over 50, and men are at higher risk than females.

49. A client has a history of major depressive disorder (MDD). Police escort the client to the ED after finding the client nude at an ATM, screaming for money to pay off credit card debt. What would make the ED psychiatrist question the client's prior diagnosis? 1) The client is experiencing symptoms of mania. 2) The client is experiencing symptoms of a severe anxiety disorder. 3) The client is experiencing symptoms of an amnestic disorder. 4) The client is experiencing symptoms of a histrionic personality disorder.

1) The client is experiencing symptoms of mania. The symptoms described in the question indicate that this client is experiencing a manic episode. Therefore, it would be appropriate for the ED psychiatrist to question the diagnosis of MDD.

Clinical course of schizophrenia

1) prodromal phase - social maladjustment, antagonistic thoughts and behavior, shy and withdrawn, poor peer relationships, doing poorly in school, antisocial behavior 2) acute illness - perceptual abnormalities, ideas of reference, suspiciousness herald onset of psychosis, deterioration in role functioning, social withdrawal, functional impairment, sleep disturbance, anxiety, irritability, depressed mood, poor concentration 3) stabilization period - chronic illness characterized by acute episodes in which symptoms are more pronounced(delusions, hallucinations, impairment in work/social relations/self care) 4) recovery - symptoms are similar to those in the prodromal phase, flat affect and impairment in role functioning are prominent 5) relapses

A nurse recognizes improvement in a client with the nursing diagnosis of ineffective role performance related to the need to perform rituals. Which behaviors indicate improvement? select all that apply: 1. the client refrains from performing rituals during stress 2. the client verbalizes using "thought stopping" when obsessive thoughts occur. 3. The client verbalizes the relationship between stress and ritualistic behaviors. 4. The client avoids stressful situations. 5. The client rationalizes ritualistic behavior. 6. The client performs ritualistic behaviors in private.

1,2,3 (refraining from performing rituals demonstrates that the client manages stress appropriately. Using "thought stopping" demonstrates the clients ability to employ appropriate interventions for obsessive thoughts. Verbalizing the relationship between stress and behaviors indicates that the client understands the disease process. Avoiding stressful situations, and rationalizing or hiding ritualistic behaviors are maladaptive methods of managing stress and anxiety)

An 8-year-old child diagnosed with OCD is admitted by the nurse to a psych facility. During the admission assessment, which behaviors would be characterized as compulsions? 1. checking and rechecking the television is turned off before going to school 2. repeatedly washing the hands 3. brushing teeth 3 times a day 4. routinely climbing up and down a flight of stairs three times before leaving the house 5. feeding the dog the same meal everyday 6. wanting to play the same video game each night

1,2,4 (compulsions involve symbolic rituals that relieve anxiety when they are performed. The disorder is caused by anxiety from obsessive thoughts and acts are seen as irrational. )

2. Which client statement is evidence of the etiology of major depressive disorder from a genetic perspective? 1. "My maternal grandmother was diagnosed with bipolar affective disorder." 2. "My mood is a 7 out of 10, and I won't harm myself or others." 3."I am so angry that my father left our family when I was 6." 4. "I just can't do anything right. I am worthless."

1. A family history of mood disorder indicates a genetic predisposition to the development of major depressive disorder. Twin, family, and adoptive studies further support a genetic link as an etiological influence in the development of mood disorders.

27. Which nursing intervention takes priority when working with a newly admitted client experiencing suicidal ideations? 1. Monitor the client at close, but irregular, intervals. 2. Encourage the client to participate in group therapy. 3. Enlist friends and family to assist the client in remaining safe after discharge. 4. Remind the client that it takes 6 to 8 weeks for antidepressants to be fully effective.

1. Clients who experience suicidal ideations must be monitored closely to prevent suicide attempts. By monitoring at irregular intervals, the nurse would prevent the client from recognizing patterns of observation. If a client recognizes a pattern of observation, the client can use the time in which he or she is not observed to plan and implement a suicide attempt.

35. Which symptoms would the nurse expect to assess in a client experiencing serotonin syndrome? 1. Confusion, restlessness, tachycardia, labile blood pressure, and diaphoresis. 2. Hypomania, akathisia, cardiac arrhythmias, and panic attacks. 3. Dizziness, lethargy, headache, and nausea. 4. Orthostatic hypotension, urinary retention, constipation, and blurred vision.

1. Confusion, restlessness, tachycardia, labile blood pressure, and diaphoresis all are symptoms of serotonin syndrome. Other symptoms include dilated pupils, loss of muscle coordination or twitching, diarrhea, headache, shivering and goose bumps. If this syndrome were suspected, the offending agent would be discontinued immediately.

10. Which symptom is an example of physiological alterations exhibited by clients diagnosed with moderate depression? 1. Decreased libido. 2. Difficulty concentrating. 3. Slumped posture. 4. Helplessness.

1. Decreased libido is a physiological alteration exhibited by clients diagnosed with moderate depression.

32. Which of the following medications may be administered before electroconvulsive therapy? Select all that apply. 1. Glycopyrrolate (Robinul). 2. Thiopental sodium (Pentothal). 3. Succinylcholine chloride (Anectine). 4. Lorazepam (Ativan). 5. Divalproex sodium (Depakote).

1. Glycopyrrolate (Robinul) is given to decrease secretions and counteract the effects of vagal stimulation induced by electroconvulsive therapy (ECT). 2. Thiopental sodium (Pentothal) is a short-acting anesthetic medication administered to produce loss of consciousness during ECT. 3. Succinylcholine chloride (Anectine) is a muscle relaxant administered to prevent severe muscle contractions during the seizure, reducing the risk for fractured or dislocated bones

16. Which nursing diagnosis takes priority for a client immediately after electroconvulsive therapy (ECT)? 1. Risk for injury R/T altered mental status. 2. Impaired social interaction R/T confusion. 3. Activity intolerance R/T weakness. 4. Chronic confusion R/T side effect of ECT.

1. Immediately after electroconvulsive therapy (ECT), risk for injury R/T altered mental status is the priority nursing diagnosis. The most common side effect of ECT is memory loss and confusion, and these place the client at risk for injury.

22. A suicidal Jewish-American client is admitted to an in-patient psychiatric unit 2 days after the death of a parent. Which intervention must the nurse include in the care of this client? 1. Allow the client time to mourn the loss during this time of shiva. 2. Distract the client from the loss and encourage participation in unit groups. 3. Teach the client alternative coping skills to deal with grief. 4. Discuss positive aspects the client has in his or her life to build on strengths.

1. In the Jewish faith, the 7-day period beginning with the burial is called shiva. During this time, mourners do not work, and no activity is permitted that diverts attention from thinking about the deceased. Because this client's parent died 2 days ago, the client needs time to participate in this religious ritual.

8. A client is admitted to an in-patient psychiatric unit with a diagnosis of major depressive disorder. Which of the following data would the nurse expect to assess? Select all that apply. 1. Loss of interest in almost all activities and anhedonia. 2. A change of more than 5% of body weight in 1 month. 3. Fluctuation between increased energy and loss of energy. 4. Psychomotor retardation or agitation. 5. Insomnia or hypersomnia.

1. Loss of interest in almost all activities and anhedonia, the inability to experience or even imagine any pleasant emotion, are symptoms of major depressive disorder (MDD). 2. Significant weight loss or gain of more than 5% of body weight in 1 month is one of the many diagnostic criteria for MDD. 4. Psychomotor retardation or agitation, occurring nearly every day, is a diagnostic criterion for MDD. 5. Sleep alterations, such as insomnia or hypersomnia, that occur nearly every day are diagnostic criteria for MDD.

1. Which nursing diagnosis supports the psychoanalytic theory of development of major depressive disorder? 1. Social isolation R/T self-directed anger. 2. Low self-esteem R/T learned helplessness. 3. Risk for suicide R/T neurochemical imbalances. 4. Imbalanced nutrition less than body requirements R/T weakness.

1. Social isolation R/T self-directed anger supports the psychoanalytic theory in the development of major depressive disorder (MDD).

37. Which of the following are examples of anticholinergic side effects from tricyclic antidepressants? Select all that apply. 1. Urinary hesitancy. 2. Constipation. 3. Blurred vision. 4. Sedation. 5. Weight gain.

1. Urinary hesitancy is an anticholinergic side effect. 2. Constipation is an anticholinergic side effect. 3. Blurred vision is an anticholinergic side effect.

25. A newly admitted client diagnosed with major depressive disorder isolates self in room and stares out the window. Which nursing intervention would be the most appropriate to implement when first establishing a nurse-client relationship? 1. Sit with the client and offer self frequently. 2. Notify the client of group therapy schedule. 3. Introduce the client to others on the unit. 4. Help the client to identify stressors of life that precipitate life crises.

1. Offering self is one technique to generate the establishment of trust with a newly admitted client diagnosed with major depressive disorder (MDD). Trust is the basis for the establishment of any nurse client relationship.

21. A client has a nursing diagnosis of risk for suicide R/T a past suicide attempt. Which outcome, based on this diagnosis, would the nurse prioritize? 1. The client will remain free from injury throughout hospitalization. 2. The client will set one realistic goal related to relationships by day 3. 3. The client will verbalize one positive attribute about self by day 4. 4. The client will be easily redirected when discussion about suicide occurs by day 5.

1. Remaining free from injury throughout hospitalization is a priority outcome for the nursing diagnosis of risk for suicide R/T a past suicide attempt. Because this outcome addresses client safety, it is prioritized.

7. Which client is at highest risk for the diagnosis of major depressive disorder? 1. A 24 year-old married woman. 2. A 64 year-old single woman. 3. A 30 year-old single man. 4. A 70 year-old married man.

1. Research indicates that depressive symptoms are highest among young, married women of low socioeconomic backgrounds. Compared with the other clients presented, this client is at highest risk for the diagnosis of major depressive disorder (MDD).

43. A client, admitted after experiencing suicidal ideations, is prescribed citalopram (Celexa). Four days later, the client has pressured speech and is noted wearing heavy makeup. What may be a potential reason for this client's behavior? 1. The client is in a manic episode caused by the citalopram (Celexa). 2. The client is showing improvement and is close to discharge. 3. The client is masking depression in an attempt to get out of the hospital. 4. The client has "cheeked" medications and taken them all at once in an attempt to overdose.

1. When an SSRI is prescribed for clients with bipolar affective disorder, it can cause alterations in neurotransmitters and trigger a hypomanic or manic episode.

A client is experiencing extrapyramidal symptoms secondary to neuroleptic drug therapy. The physician ordered biperiden (Akineton), 2mg tid IV. If a 5mg/mL vial is used, what is the total amount, in mL per day, that the nurse will administer?

1.2

A client has been taking lithium for several years with good symptom control. The client presents in the emergency department with blurred vision, tinnitus, and severe diarrhea. The nurse should correlate these symptoms with which lithium level?

1.7 mEq/L

A client has been taking lithium for several years with good symptom control. The client presents in the emergency department with blurred vision, tinnitus, and severe diarrhea. Which lithium level should the nurse correlate with these symptoms? A. 1.3 mEq/L B. 1.7 mEq/L C. 2.3 mEq/L D. 3.7 mEq/L

1.7 mEq/L

Tony, age 21, has been diagnosed with schizophrenia. He has been socially isolated and hearing voices telling him to kill his parents. He has been admitted to the psychiatric unit from the emergency department. The initial nursing intervention for Tony is to: A. give him an injection of Thorazine B. ensure a safe environment for him and others C. Place him in restraints. D. Order him a nutritious diet

B. ensure a safe environment for him and others

54. Chloe is suffering from depression and not responding to antidepressant treatment. She asks the nurse to tell her more about transcranial magnetic stimulation (TMS). Which of the following responses is accurate with regard to this treatment modality? 1) TMS uses magnetic energy to induce a seizure. 2) One study concluded that electroconvulsive therapy was more effective than TMS for short-term treatment of depression. 3) TMS is a safe and inexpensive treatment for depression. 4) TMS has been demonstrated to be more effective than any other treatment modality for depression.

2) One study concluded that electroconvulsive therapy was more effective than TMS for short-term treatment of depression. Patients often rely on nurses to provide current accurate information about new or experimental treatment modalities, so it is important for nurses to continue to evaluate current evidence in order to provide patients with the most up-to-date, accurate information.

55. When teaching about the tricyclic group of antidepressant medications, which information should the nurse include? 1) Strong or aged cheese should not be eaten while the client is taking this group of medications. 2) The full therapeutic potential of tricyclics may not be reached for 4 weeks. 3) Tricyclics may cause hypomania or recent memory impairment. 4) Tricyclics should not be given with antianxiety agents.

2) The full therapeutic potential of tricyclics may not be reached for 4 weeks.

45. The physician orders fluoxetine (Prozac) for a client diagnosed with depression. Which information is true about this medication? 1) Prozac is a tricyclic antidepressant. 2) The therapeutic effect of Prozac occurs 2 to 4 weeks after treatment is begun. 3) Aged cheese, yogurt, soy sauce, and bananas should not be eaten while the client is taking this drug. 4) Prozac may be administered in combination with monoamine oxidase inhibitors (MAOIs).

2) The therapeutic effect of Prozac occurs 2 to 4 weeks after treatment is begun.

30. A nursing instructor is presenting statistics regarding suicide. Which student statement indicates that learning has occurred? 1. "Approximately 10,000 individuals in the United States will commit suicide each year." 2. "Almost 95% of all individuals who commit or attempt suicide have a diagnosed mental disorder." 3."Suicide is the eighth leading cause of death among young Americans 15 to 24 years old." 4. "Depressive disorders account for 70% of all individuals who commit or attempt suicide."

2. Almost 95% of all individuals who commit or attempt suicide have a diagnosed mental disorder. Most suicides are associated with mood disorders.

20. Which client would the charge nurse assign to an agency nurse working on the in-patient psychiatric unit for the first time? 1. A client experiencing passive suicidal ideations with a past history of an attempt. 2. A client rating mood as 3/10 and attending but not participating in group therapy. 3. A client lying in bed all day long in a fetal position and refusing all meals. 4. A client admitted for the first time with a diagnosis of major depression.

2. Although this client rates mood low, there is no indication of suicidal ideations, and the client is attending groups in the milieu. Because this client is observable in the milieu by all staff members, assignment to an agency nurse would be appropriate.

11. Major depressive disorder would be most difficult to detect in which of the following clients? 1. A 5 year-old girl. 2. A 13 year-old boy. 3. A 25 year-old woman. 4. A 75 year-old man.

2. Assessment of depressive disorders in 13 year-old children would include feelings of sadness, loneliness, anxiety, and hopelessness. These symptoms may be perceived as normal emotional stresses of growing up.

33. A client diagnosed with major depressive disorder is prescribed phenelzine (Nardil). Which teaching should the nurse prioritize? 1. Remind the client that the medication takes 6 to 8 weeks to take full effect. 2. Instruct the client and family about the many food-drug and drug-drug interactions. 3. Teach the client about the possible sexual side effects and insomnia that can occur. 4. Educate the client about taking the medication prescribed even after symptoms improve.

2. Because there are numerous drug-food and drug drug interactions that may precipitate a hypertensive crisis during treatment with MAOIs, it is critical that the nurse prioritize this teaching.

40. A client has been taking bupropion (Wellbutrin) for more than 1 year. The client has been in a car accident with loss of consciousness and is brought to the ED. For which reason would the nurse question the continued use of this medication? 1. The client may have a possible injury to the gastrointestinal system. 2. The client is at risk for seizures from a potential closed head injury. 3. The client is at increased risk of bleeding while taking bupropion. 4. The client may experience sedation from bupropion, making assessment difficult.

2. Bupropion lowers the seizure threshold. Bupropion is contraindicated for clients who have increased potential for seizures, such as a client with a closed head trauma injury.

34. A client diagnosed with major depressive disorder is newly prescribed sertraline (Zoloft). Which of the following teaching points would the nurse review with the client? Select all that apply. 1. Ask the client about suicidal ideations related to depressed mood. 2. Discuss the need to take medications, even when symptoms improve. 3. Instruct the client about the risks of abruptly stopping the medication. 4. Alert the client to the risks of dry mouth, sedation, nausea, and sexual side effects. 5. Remind the client that the medication's full effect may not occur for 6 to 8 weeks.

2. Discussing the need for medication compliance, even when symptoms improve, is a teaching point that the nurse would need to review with a client who is newly prescribed sertraline. 3. Instructing the client about the risk for discontinuation syndrome is a teaching point that the nurse would need to review with a client who is newly prescribed sertraline. 4. Alerting the client to the risks of dry mouth, sedation, nausea, and sexual side effects is a teaching point that the nurse would need to review with a client who is newly prescribed sertraline. 5. Reminding the client that sertraline's full effect may not occur for 6 to 8 weeks is a teaching point that the nurse would need to review with a client who is newly prescribed sertraline.

39. Which situation would place a client at high risk for a life-threatening hypertensive crisis? 1. A client is prescribed tranylcypromine (Parnate) and eats chicken salad. 2. A client is prescribed isocarboxazid (Marplan) and drinks hot chocolate. 3. A client is prescribed venlafaxine (Effexor) and drinks wine. 4. A client is prescribed phenelzine (Nardil) and eats fresh roasted chicken.

2. Isocarboxazid is an MAOI, and the intake of chocolate would cause a life-threatening hypertensive crisis.

23. A client denying suicidal ideations comes into the emergency department complaining about insomnia, irritability, anorexia, and depressed mood. Which intervention would the nurse implement first? 1. Request a psychiatric consultation. 2. Complete a thorough physical assessment including lab tests. 3. Remove all hazardous materials from the environment. 4. Place the client on a one-to-one observation.

2. Numerous physical conditions can contribute to symptoms of insomnia, including irritability, anorexia, and depressed mood. It is important for the nurse to rule out these physical problems before assuming that the symptoms are psychological in nature.

18. A client's outcome states, "The client will make a plan to take control of one life situation by discharge." Which nursing diagnosis documents the client's problem that this outcome addresses? 1. Impaired social interaction. 2. Powerlessness. 3. Knowledge deficit. 4. Dysfunctional grieving.

2. Powerlessness is defined as the perception that one's own action would not significantly affect an outcome—a perceived lack of control over a current situation or immediate happening.

42. A client recently prescribed venlafaxine (Effexor) 37.5 mg bid complains of dry mouth, orthostatic hypotension, and blurred vision. Which nursing intervention is appropriate? 1. Hold the next dose, and document symptoms immediately. 2. Reassure the client that side effects are transient, and teach ways to deal with them. 3. Call the physician to receive an order for benztropine (Cogentin). 4. Notify the dietary department about restrictions related to monoamine oxidase inhibitors.

2. The nurse needs to teach the client about acceptable side effects, and what the client can do to deal with them. The nurse can suggest that the client use ice chips, sip small amounts of water, or chew sugar-free gum or candy to moisten the dry mouth. For orthostatic hypotension, the nurse may encourage the client to change positions slowly. For blurred vision, the nurse may encourage the use of moisturizing eyedrops.

When a client suddenly becomes aggressive and violent on the unit, which of the following approaches would be best for the nurse to use first? A. Provide large motor activities to relieve the client's pent-up tension. B. Administer a dose of PRN chlorpromazine to keep the client calm. C. Call for sufficient help to control the situation safely. D. Convey to the client that his behavior is unacceptable and will not be permitted.

C. Call for sufficient help to control the situation safely.

14. A nurse is planning to teach about appropriate coping skills. The nurse would expect which client to be at the highest level of readiness to participate in this instruction? 1. A newly admitted client with an anxiety level of 8/10 and racing thoughts. 2. A client admitted 6 days ago for a manic episode refusing to take medications. 3. A newly admitted client experiencing suicidal ideations with a plan to overdose. 4. A client admitted 6 days ago for suicidal ideations following a depressive episode.

4. A client admitted 6 days ago for suicidal ideations has begun to stabilize because of the treatment received during this time frame.

59. Lamont has been scheduled for electroconvulsive therapy (ECT) and asks the nurse, "Is it true what I heard, that ECT causes brain damage?" Which of these would be the most appropriate, evidence-based response by the nurse? 1) "ECT has no effect on brain function at all." 2) "ECT has only been shown to cause brain damage in the elderly population." 3) "There is no evidence that ECT causes permanent changes in brain structure or function." 4) "Current evidence suggests that brain damage after ECT treatments is related to the anesthetic agents, not the treatment itself."

3) "There is no evidence that ECT causes permanent changes in brain structure or function."

57. Ursula has sought counseling for persistent depressive disorder. She identifies that she has "always had low self-esteem" and says "I just let people walk all over me." The nurse is providing psycho-educational groups on improving self-esteem. Ursula would likely benefit from education on which of the following topics? 1) Antipsychotic medications 2) Anger management 3) Assertive communication 4) Alcoholics Anonymous groups

3) Assertive communication Education in assertive communication is recognized as an intervention to build positive self-esteem. Ursula's statement that she lets people walk all over her is an indication that this would be beneficial education for her.

47. Hannah is being evaluated for postpartum depression after she reported to her family physician that she just doesn't think she can take care of her baby. She expresses fear that God will take her children from her for being a bad mother. Which of the following is the highest priority for the nurse to assess during the initial interview? 1) The number of children Hannah is currently trying to care for. 2) Availability of support systems in Hannah's family. 3) Risks for suicide and/or infanticide. 4) What time of day the symptoms occur.

3) Risks for suicide and/or infanticide. The risks for suicide and/or infanticide should not be overlooked. Hannah's concern that she can't care for the baby and that God might take her children raises additional concern that further assessment for these risks is a priority.

26. A client diagnosed with major depressive disorder is being considered for electroconvulsive therapy (ECT). Which client teaching should the nurse prioritize? 1. Empathize with the client about fears regarding ECT. 2. Monitor for any cardiac alterations to prevent possible negative outcomes. 3. Discuss with the client and family expected short-term memory loss. 4. Inform the client that injury related to induced seizure commonly occurs.

3. An expected and acceptable side effect of ECT is short-term memory loss. It is important for the nurse to teach the client and family members this information to prevent unnecessary anxiety about this symptom.

12. Which is the key to understanding whether a child or adolescent is experiencing an underlying depressive disorder? 1. Irritability with authority. 2. Being uninterested in school. 3. A change in behaviors over a 2-week period. 4. Feeling insecure at a social gathering.

3. Change in behavior is an indicator that differentiates mood disorders from the typical stormy behaviors of adolescence.

6. Which charting entry most accurately documents a client's mood? 1. "The client expresses an elevation in mood." 2. "The client appears euthymic and is interacting with others." 3. "The client isolates self and is tearful most of the day." 4."The client rates mood at a 2 out of 10."

4. The use of a mood scale objectifies the subjective symptom of mood as a pain scale objectifies the subjective symptom of pain. The use of scales is the most accurate way to assess subjective data.

17. A newly admitted client diagnosed with major depressive disorder has a history of two suicide attempts by hanging. Which nursing diagnosis takes priority? 1. Risk for violence directed at others R/T anger turned outward. 2. Social isolation R/T depressed mood. 3. Risk for suicide R/T history of attempts. 4. Hopelessness R/T multiple suicide attempts.

3. Risk for suicide R/T history of attempts is a priority nursing diagnosis.

19. A client admitted with major depressive disorder has a nursing diagnosis of ineffective sleep pattern R/T aches and pains. Which is an appropriate short-term outcome for this client? 1. The client will express feeling rested upon awakening. 2. The client will rate pain level at or below a 4/10. 3. The client will sleep 6 to 8 hours at night by day 5. 4. The client will maintain a steady sleep pattern while hospitalized.

3. The appropriate short-term outcome for the nursing diagnosis of ineffective sleep pattern R/T aches and pains is to expect the client to sleep 6 to 8 hours a night by day 5. This outcome is client specific, realistic, and measurable and includes a time frame.

5. A client plans and follows through with the wake and burial of a child lost in an automobile accident. Using Engel's model of normal grief response, in which stage would this client fall? 1. Resolution of the loss. 2. Recovery. 3. Restitution. 4. Developing awareness.

3. The client in the question is exhibiting signs associated with Engel's stage of restitution.

9. A client is exhibiting behavioral symptoms of depression. Which charting entry would appropriately document these symptoms? 1. "Rates mood as 4/10." 2. "Expresses thoughts of poor self-esteem during group." 3. "Became irritable and agitated on waking." 4. "Rates anxiety as 2/10 after receiving lorazepam (Ativan)."

3. When the client becomes irritable and agitated on awakening, the client is exhibiting behavioral symptoms of depression. Other behavioral symptoms include, but are not limited to, tearfulness, restlessness, slumped posture, and withdrawal.

31. A client diagnosed with major depressive disorder has an outcome that states, "The client will verbalize a measure of hope about the future by day 3." Which client statement indicates this outcome was successful? 1. "I don't want to die because it would hurt my family." 2. "I need to go to group and get out of this room." 3."I think I am going to talk to my boss about conflicts at work." 4. "I thank you for your compassionate care."

3. When the client begins to plan how to deal with conflicts at work, the client is focusing on a hopeful future. This indicates that the outcome of verbalizing a measure of hope about the future by day 3 has been successful.

50. Shelly is a patient on the inpatient psychiatric unit and was diagnosed with major depressive disorder. She is staying in her room and sleeping most of the day. Which of the following approaches by the nurse would best facilitate getting Shelly involved in the occupational therapy group on the unit? 1) "Would you like to go to occupational therapy? It is starting right now." 2) "Let me know what activities you want to be involved in and I'll give you a schedule." 3) "If you don't go to occupational therapy today, you will have to stay in your room for the entire evening." 4) "Occupational therapy is starting in 30 minutes; I'll help you get ready."

4) "Occupational therapy is starting in 30 minutes; I'll help you get ready." This response by the nurse uses an active approach (stating the expected behavior rather than encouraging the patient to decide), provides time to prepare, and offers assistance in the process. This approach would most likely facilitate Shelly's participation in the occupational therapy activities.

60. A client is being treated with sertraline (Zoloft) for major depression. The client tells the nurse, "I've been taking this drug for only a week, but I'm sleeping better and my appetite has improved." Which is the most appropriate response by the nurse? 1) "It will take a minimum of 3 to 4 weeks for therapeutic effects to occur." 2) "Sleep disturbances and appetite problems are not affected by Zoloft." 3) "A change in your environment and activity is the reason for this improvement." 4) "The initiation of Zoloft therapy can improve insomnia and appetite within 1 week."

4) "The initiation of Zoloft therapy can improve insomnia and appetite within 1 week." Zoloft is known to improve middle and terminal insomnia, appetite disturbances, and anxiety as early as 1 week after initiation of treatment.

46. Tara experienced the death of a parent 2 years ago. She has not been able to work since the death, cannot look at any of the parent's belongings, and cries daily for hours at a time. Which nursing diagnosis most accurately describes Tara's problem? 1) Post-trauma syndrome R/T parent's death. 2) Anxiety R/T parent's death. 3) Coping, ineffective, R/T parent's death. 4) Grieving, complicated, R/T parent's death.

4) Grieving, complicated, R/T parent's death. The excessive reactions that the individual continues to exhibit, such as daily crying, inability to return to work, and inability to look at the parent's belongings after a 2-year period, are indicative of dysfunctional or complicated grieving. This individual's grieving response has arrested in the anger stage of grief and is manifested by exaggerated grieving behaviors.

29. A client has a nursing diagnosis of dysfunctional grieving R/T loss of a job AEB inability to seek employment because of sad mood. Which would support a resolution of this client's problem? 1. The client reports an anxiety level of 2 out of 10 and denies suicidal ideations. 2. The client exhibits trusting behaviors toward the treatment team. 3. The client is noted to be in the denial stage of the grief process. 4. The client recognizes and accepts the role he or she played in the loss of the job.

4. Accepting responsibility for the role played in a loss indicates that the client has moved forward in the grieving process and resolved the problem of dysfunctional grieving.

38. A client diagnosed with major depressive disorder and experiencing suicidal ideation is showing signs of anxiety. Alprazolam (Xanax) is prescribed. Which assessment should be prioritized? 1. Monitor for signs and symptoms of physical and psychological withdrawal. 2. Teach the client about side effects of the medication, and how to handle these side effects. 3. Assess for nausea, and give the medication with food if nausea occurs. 4. Ask the client to rate his or her mood on a mood scale, and monitor for suicidal ideations.

4. Alprazolam is a central nervous system depressant, and it is important for the nurse in this situation to monitor for worsening depressive symptoms and possible worsening of suicidal ideations.

4. Which statement describes a major difference between a client diagnosed with major depressive disorder and a client diagnosed with dysthymic disorder? 1. A client diagnosed with dysthymic disorder is at higher risk for suicide. 2. A client diagnosed with dysthymic disorder may experience psychotic features. 3. A client diagnosed with dysthymic disorder experiences excessive guilt. 4. A client diagnosed with dysthymic disorder has symptoms for at least 2 years.

4. An individual suspected to have dysthymic disorder needs to experience symptoms for at least 2 years before a diagnosis can be made. The essential feature is a chronically depressed mood (or possibly an irritable mood in children and adolescents) for most of the day, more days than not, for at least 2 years (1 year for children and adolescents). Clients with a diagnosis of MDD show impaired social and occupational functioning that has existed for at least 2 weeks.

24. A client diagnosed with major depressive disorder has a nursing diagnosis of low self-esteem R/T negative view of self. Which cognitive nursing intervention would be appropriate to deal with this client's problem? 1. Promote attendance in group therapy to assist the client in socializing. 2. Teach assertiveness skills by role-playing situations. 3. Encourage the client to journal to uncover underlying feelings. 4. Focus on strengths and accomplishments to minimize failures.

4. Focusing on strengths and accomplishments to minimize failures is a cognitive nursing intervention. Cognitive interventions focus on altering distortions of thoughts and negative thinking.

41. A client experiencing suicidal ideations with a plan to overdose on medications is admitted to an in-patient psychiatric unit. Mirtazapine (Remeron) is prescribed. Which nursing intervention takes priority? 1. Remind the client that medication effectiveness may take 2 to 3 weeks. 2. Teach the client to take the medication with food to avoid nausea. 3. Check the client's blood pressure every shift to monitor for hypertension. 4. Monitor closely for signs that the client might be "cheeking" medications.

4. If a client comes into the in-patient psychiatric unit with a plan to overdose, it is critical that the nurse monitor for cheeking and hoarding of medications. Clients may cheek and hoard medications to take, as an overdose, at another time.

28. A nursing student is studying major depressive disorder. Which student statement indicates that learning has occurred? 1. "1% of the population is affected by depression yearly." 2. "2% to 5% of women experience depression during their lifetimes." 3. "1% to 3% of men become clinically depressed." 4."Major depression is a leading cause of disability in the United States."

4. Major depression is one of the leading causes of disability in the United States.

36. Which medication would be classified as a tricyclic antidepressant? 1. Bupropion (Wellbutrin). 2. Mirtazapine (Remeron). 3. Citalopram (Celexa). 4. Nortriptyline (Pamelor).

4. Nortriptyline (Pamelor) is classified as a tricyclic antidepressant. Other tricyclic antidepressants include amitriptyline (Elavil), doxepin (Sinequan), and imipramine (Tofranil).

3. During an intake assessment, which client statement is evidence of the etiology of major depressive disorder from an object-loss theory perspective? 1. "I am so angry all the time and seem to take it out on myself." 2. "My grandmother and great-grandfather also had depression." 3. "I just don't think my life is ever going to get better. I keep messing up." 4. "I don't know about my biological family; I was in foster care as an infant."

4. Object-loss theorists suggest that depressive illness occurs as a result of being abandoned by or otherwise separated from a significant other during the first 6 months of life. The client in the question experienced parental abandonment, and according to objectloss theory, this loss has led to the diagnosis of MDD.

Which nursing intervention would be most appropriate when caring for an acutely agitated client with paranoia? A. Provide neon lights and soft music. B. Maintain continual eye contact throughout the interview. C. Use therapeutic touch to increase trust and rapport. D. Provide personal space to respect the clients boundaries.

ANS: D The most appropriate nursing intervention is to provide personal space to respect the clients boundaries. Providing personal space may serve to reduce anxiety and thus reduce the clients risk for violence.

A client diagnosed with Bipolar Disorder states, "I hate oatmeal. Let's get everybody together to do exercises. I'm thirsty and I'm burning up. Get out of my way; I have to see that guy." Which is the priority nursing action? A. Assess the client's vital signs. B. Offer to have the dietitian discuss food preferences. C. Encourage the client to lead the exercise program in the community meeting. D. Acknowledge the client briefly and then walk away.

Assess the client's vital signs.

The family of a patient who has been prescribed antipsychotic medication tells the nurse they understand there are potentially fatal side effects with these medications. They ask the nurse for information about what they should look for that could signal potentially dangerous or fatal side effects. Which of the following responses by the nurse are accurate with regard to the family's question? (Select all that apply) A. "If the patient has acute muscle spasms or the patient's eyes appear to be rolling back, emergency intervention should be sought." B. "if the patient has an unusually high fever and complains of muscle rigidity, any further antipsychotic medication should be discontinued and immediate emergency intervention should be sought." C. "If the patient complains of sore throat, fever, and malaise, the doctor should be contacted to evaluate for a possible dangerous side effect of the mediation." D. "if the male patient begins to show signs of breast enlargement or the female patient experiences amenorrhea, take the patient immediately to the ER." E. "If the patient's psychotic symptoms appear to be absent, call the doctor immediately."

A. "If the patient has acute muscle spasms or the patient's eyes appear to be rolling back, emergency intervention should be sought." B. "if the patient has an unusually high fever and complains of muscle rigidity, any further antipsychotic medication should be discontinued and immediate emergency intervention should be sought." C. "If the patient complains of sore throat, fever, and malaise, the doctor should be contacted to evaluate for a possible dangerous side effect of the mediation." Feedback 1: These symptoms are indicative of an acute dystonia, which can progress to laryngospasm if not treated. Emergency intervention with an anticholinergic such as Cogentin is needed to reverse this side effect. Feedback 2: These symptoms are indicative of neuroleptic malignant syndrome, which can progress rapidly and be fatal. Immediate discontinuation of antipsychotic medication and emergency intervention are critical needs. Feedback 3: These symptoms may be indicative of agranulocytosis, which can be fatal is not treated. Further bloodwork is needed.

A client diagnosed with schizophrenia states, "My roommate is plotting to have others kill me." Which is the appropriate nursing response? A. "i know you believe that to be true, but i find that hard to believe." B. "What would make you think such a thing?" C. "I know your roommate. He would do no such thing." D. "I can see why you feel that way."

A. "i know you believe that to be true, but i find that hard to believe." This client is experiencing a persecutory delusion. this nursing response is an example o voicing doubt, which expresses uncertainty as to the reality of the client's perceptions. this is an appropriate therapeutic communication technique in dealing with clients experiencing delusional thinking.

Which of the following instructions regarding lithium therapy should be included in the nurse's discharge teaching? Select all that apply. A. Avoid excessive use of beverages containing caffeine. B. Maintain a consistent sodium intake. C. Consume at least 2500 to 3000 mL of fluid per day. D. Restrict sodium content. E. Restrict fluids to 1500 mL per day.

A. Avoid excessive use of beverages containing caffeine. B. Maintain a consistent sodium intake. C. Consume at least 2500 to 3000 mL of fluid per day.

Clint, a client on the psychiatric unit, has been diagnosed with schizophrenia. He begins to tell the nurse about how the CIA is looking for him and will kill him if they find him. Clint's belief is an example of a: A. Delusion of persecution B. Delusion of reference C. Delusion of control or influence D. Delusion of grandeur

A. Delusion of persecution

A client diagnosed with schizophrenia experiences identity confusion and communicates with the nurse using echolalia. What is the client attempting to do by using this form of speech? A. Identify with the person speaking B. Imitate the nurse's movements C. Alleviate alogia D. Alleviate avolition

A. Identify with the person speaking Echolalia is a parrot-like repetition of overheard words or fragments of speech. It is an attempt by the client to identify with the person who is speaking.

A client and nurse therapist are developing a treatment plan that includes strategies to manage bipolar disorder. Which should be included? Select all that apply. A. Maintain a consistent sleep schedule. B. Become an expert on mental health. C. Create a daily medication schedule. D. Set a time frame to achieve remission. E. Develop an emergency plan.

A. Maintain a consistent sleep schedule. B. Become an expert on mental health. C. Create a daily medication schedule. E. Develop an emergency plan.

The clinic nurse is reviewing the medication list of a client diagnosed with Medication-Induced Bipolar Disorder. The nurse recognizes which may have precipitated the client's mood disturbance? Select all that apply. A. Oral contraceptives B. Antihypertensives C. Dopamine agonist D. Corticosteroids E. Alpha-adrenergics

A. Oral contraceptives B. Antihypertensives D. Corticosteroids

Which of the following explanations should the nurse include when teaching parents why it is difficult to diagnose a child or adolescent exhibiting symptoms of bipolar disorder? Select all that apply. A. Symptoms of bipolar disorder are similar to those of attention deficit-hyperactivity disorder. B. Children are naturally active, energetic, and spontaneous. C. Neurotransmitter levels vary considerably in accordance with age. D. The diagnosis of Bipolar Disorder cannot be assigned prior to the age of 18. E. Genetic predisposition is not a reliable diagnostic determinant.

A. Symptoms of bipolar disorder are similar to those of attention deficit-hyperactivity disorder. B. Children are naturally active, energetic, and spontaneous.

A client diagnosed with bipolar disorder states, "I hate oatmeal. Let's get everybody together to do exercises. I'm thirsty and I'm burning up. Get out of my way; I have to see that guy." What should be the priority nursing action?

Assess the client's vital signs.

A client who is experiencing command hallucinations is hospitalized after jumping from a bridge. The client's parents insist that their son feel rather than jumped. Which of the following likely explain the parents' response? (Select all that apply) A. The parents are in denial about the reality of their son's mental illness B. The parents are grieving over the loss of their expectations for their child C. The parents do not understand the extent or seriousness of mental illness D. The parents reject the idea of their son having a mental illness E. The parents are showing support for their son.

A. The parents are in denial about the reality of their son's mental illness B. The parents are grieving over the loss of their expectations for their child C. The parents do not understand the extent or seriousness of mental illness D. The parents reject the idea of their son having a mental illness Feedback 1: by stating the jump was a fall, the parents are expressing denial and minimizing the problem Feedback 2: The child's attempted suicide could generate a loss of hope that their child will meet parental expectations. This can occur any time a child is physically or mentally different. Feedback 3: The parents may have a knowledge deficit and truly may not understand the implications of their child's mental illness Feedback 4: By claiming that their son feel rather than jumped from the bridge, the parents are embracing an accidental cause and rejecting the possibility of mental illness.

The family of a client diagnosed with schizophrenia tells the nurse that they were at a NAMI meeting and heard that the recovery model for intervention with people with schizophrenia is gaining recognition as a desirable approach. They ask the nurse to describe this model. Which of these responses by the nurse are accurate statements about the recovery model? (Select all that apply) A. This model supports that recovery is an obtainable objective for people with schizophrenia B. This approach engages the client in an Alcoholics Anonymous (AA) - like 12-step program for recovery C. the recovery model actively engages the client in determining the goals for the treatment plan. D. The recovery model should not be confused with providing a "cure" for schizophrenia E. The recovery model is controversial because it stigmatizes the person with schizophrenia

A. This model supports that recovery is an obtainable objective for people with schizophrenia C. the recovery model actively engages the client in determining the goals for the treatment plan. D. The recovery model should not be confused with providing a "cure" for schizophrenia Feedback 1: Conventional models for treatment in schizophrenia have been criticized for potentially inhibiting a client's ability to recognize his or her potential because they focus too heavily on the disease as one in which recovery is not obtainable. The recovery model shifts the focus toward recovery as an attainable goal. Feedback 3: Central to the recovery model in intervention with people with schizophrenia is a patient-centered approach in which the clinician and the client work together to develop a treatment plan that is in alignment with goals set forth by the client Feedback 4: It is important in educating clients and families that the recovery model is not to be confused with promising a remission or a cure for this illness. Instead, it focuses on potential to function more autonomously rather than a primary focus on managing an intractable illness.

A patient admitted to the psychiatric unit and diagnosed with schizophrenia reports to the nurse that there are people playing drums in his chest. Which of these would be appropriate interventions by the nurse? (Select all that apply) A. check the patient's vital signs B. tell the patient that these are tactile hallucinations and that he need not be concerned C. Ask the patient to describe more completely what he is feeling D. Give the patient PRN Cogentin as ordered E. Encourage the patient to discuss this with the music therapist.

A. check the patient's vital signs C. Ask the patient to describe more completely what he is feeling Feedback 1: This intervention is a priority to ensure that the patient's symptoms are not secondary to a medical emergency such as heart attack. Feedback 3: This is an appropriate intervention since further assessment is needed to ensure that the patient's physiological needs are being met.

A client diagnosed with schizophrenia experiences identify confusion and communicates with the nurse using echolalia. What is the client attempting to do by using this form of speech? A. identify with the person speaking B. imitate the nurse's movements C. alleviate alogia D. alleviate avolition

A. identify with the person speaking Echolalia is a parrot-like repetition of overheard words or fragments of speech. it is an attempt by the client to identify with the person who is speaking.

A withdrawn client, newly diagnosed with schizophrenia, is experiencing delusional thinking. Which nursing intervention is most appropriate? A. present objective reality B. use self-disclosure C. use physical touch for reassurance D. provide an in-depth explanation of unit rules and regulations

A. present objective reality When communicating with a client diagnosed with schizophrenia, the nurse should reinforce and focus on reality by talking about real events and real people. Discussions that focus on false ideas reinforce the client's delusions.

A client on an inpatient unit is diagnosed with Bipolar Disorder: Manic Episode. During a discussion in the dayroom about weekend activities, the client raises his voice, becomes irritable, and insists that plans change. Which should be the nurse's initial intervention? A. Ask the group to take a vote on alternative weekend events. B. Remind the client to quiet down or leave the dayroom. C. Assist the client to move to a calmer location. D. Discuss with the client impulse control problems.

Assist the client to move to a calmer location.

How would a nurse differentiate a client diagnosed with obsessive-compulsive disorder (OCD) from a client diagnosed with obsessive-compulsive personality disorder? A. Clients diagnosed with OCD experience both obsessions and compulsions, and clients diagnosed with obsessive-compulsive personality disorder do not. B. Clients diagnosed with obsessive-compulsive personality disorder experience both obsessions and compulsions, and clients diagnosed with OCD do not. C. Clients diagnosed with obsessive-compulsive personality disorder experience only obsessions, and clients diagnosed with OCD experience only compulsions. D. Clients diagnosed with OCD experience only obsessions, and clients diagnosed with obsessive-compulsive personality disorder experience only compulsions.

ANS: A A client diagnosed with OCD experiences both obsessions and compulsions. Clients diagnosed with obsessive-compulsive personality disorder exhibit a pervasive pattern of preoccupation with orderliness, perfectionism, and mental and interpersonal control.

A client is admitted to the psychiatric unit with a diagnosis of major depression. The client is unable to concentrate, has no appetite, and is experiencing insomnia. Which should be included in this client's plan of care? A. A simple, structured daily schedule with limited choices of activities B. A daily schedule filled with activities to promote socialization C. A flexible schedule that allows the client opportunities for decision making D. A schedule that includes mandatory activities to decrease social isolation

ANS: A A client diagnosed with depression has difficulty concentrating and may be overwhelmed by activity overload or the expectation of independent decision making. A simple, structured daily schedule with limited choices of activities is more appropriate.

A client diagnosed with major depressive disorder was raised in an excessively religiously based household. Which nursing intervention would be most appropriate to address this client's underlying problem? A. Encourage the client to bring into awareness underlying sources of guilt. B. Teach the client that religious beliefs should be put into perspective throughout the life span. C. Confront the client with the irrational nature of the belief system. D. Assist the client to modify his or her belief system in order to improve coping skills.

ANS: A A client raised in an excessively religiously based household maybe at risk for experiencing guilt to the point of accepting liability in situations for which one is not responsible. The client may view himself or herself as evil and deserving of punishment leading to depression. Assisting the client to bring these feelings into awareness allows the client to realistically appraise distorted responsibility and dysfunctional guilt.

A nurse reviews the laboratory data of a client suspected of having major depressive disorder. Which laboratory value would potentially rule out this diagnosis? A. Thyroid-stimulating hormone (TSH) level of 6.2 U/mL B. Potassium (K+) level of 4.2 mEq/L C. Sodium (Na+) level of 140 mEq/L D. Calcium (Ca2+) level of 9.5 mg/dL

ANS: A According to the DSM-IV-TR, symptoms of major depressive disorder cannot be due to the direct physiological effects of a general medical condition (e.g., hypothyroidism). The diagnosis of major depressive disorder may be ruled out if the client's laboratory results indicate a high TSH level which results from a low thyroid function or hypothyroidism. In hypothyroidism, metabolic processes are slowed leading to depressive symptoms.

Arthur, who is diagnosed with obsessive-compulsive disorder, reports to the nurse that he cant stop thinking about all the potentially life threatening germs in the environment. What is the most accurate way for the nurse to document this symptom? A. Patient is expressing an obsession with germs. B. Patient is manifesting compulsive thinking. C. Patient is expressing delusional thinking about germs. D. Patient is manifesting arachnophobia of germs.

ANS: A Obsessions are unwanted, intrusive, repetitive thoughts. Compulsions are unwanted, repetitive behavior patterns in response to obsessive thoughts that are efforts to reduce anxiety.

A client is admitted with a diagnosis of persistent depressive disorder. Which client statement would describe a symptom consistent with this diagnosis? A. I am sad most of the time and Ive felt this way for the last several years. B. I find myself preoccupied with death. C. Sometimes I hear voices telling me to kill myself. D. Im afraid to leave the house.

ANS: A Persistent depressive disorder is characterized by depressed mood for most of day, for more days than not, for at least 2 years. Thoughts of death would be more consistent with major depressive disorder; hearing voices is more consistent with a psychotic disorder; and fear of leaving the house is more consistent with a phobia.

A nursing diagnosis of ineffective coping R/T feelings of loneliness AEB bingeing then purging when alone, is assigned to a client diagnosed with bulimia nervosa. Which is an appropriate outcome related to this nursing diagnosis? A. The client will identify two alternative methods of dealing with isolation by day 3. B. The client will appropriately express angry feelings about lack of control by week 2. C. The client will verbalize two positive self attributes by day 3. D. The client will list five ways that the body reacts to bingeing and purging.

ANS: A The ability to identify alternative methods of dealing with isolation will provide the client with effective coping strategies to use instead of bingeing and purging.

A nurse is providing discharge teaching to a client taking a benzodiazepine. Which client statement would indicate a need for further follow-up instructions? A. "I will need scheduled blood work in order to monitor for toxic levels of this drug." B. "I won't stop taking this medication abruptly because there could be serious complications." C. "I will not drink alcohol while taking this medication." D. "I won't take extra doses of this drug because I can become addicted."

ANS: A The client indicates a need for additional information about taking benzodiazepines when stating the need for blood work to monitor for toxic levels. No blood work is needed when taking a short-acting benzodiazepine. The client should understand that taking extra doses of a benzodiazepine may result in addiction and that the drug should not be taken in conjunction with alcohol.

A client diagnosed with panic disorder states, "When an attack happens, I feel like I am going to die." Which is the most appropriate nursing reply? A. "I know it's frightening, but try to remind yourself that this will only last a short time." B. "Death from a panic attack happens so infrequently that there is no need to worry." C. "Most people who experience panic attacks have feelings of impending doom." D. "Tell me why you think you are going to die every time you have a panic attack."

ANS: A The most appropriate nursing reply to the client's concerns is to empathize with the client and provide encouragement that panic attacks last only a short period. Panic attacks usually last minutes but can, rarely, last hours. Symptoms of depression are also common with this disorder.

A nurse observes dental deterioration when assessing a client diagnosed with bulimia nervosa. What explains this assessment finding? A. The emesis produced during purging is acidic and corrodes the tooth enamel. B. Purging causes the depletion of dietary calcium. C. Food is rapidly ingested without proper mastication. D. Poor dental and oral hygiene leads to dental caries.

ANS: A The nurse recognizes that dental deterioration has resulted from the acidic emesis produced during purging that corrodes the tooth enamel. Excessive vomiting may also lead to dehydration and electrolyte imbalance.

A client has a history of excessive fear of water. What is the term that a nurse should use to describe this specific phobia, and under what subtype is this phobia identified? A. Aquaphobia, a natural environment type of phobia B. Aquaphobia, a situational type of phobia C. Acrophobia, a natural environment type of phobia D. Acrophobia, a situational type of phobia

ANS: A The nurse should determine that an excessive fear of water is identified as aquaphobia which is a natural environment type of phobia. Natural environment-type phobias are fears about objects or situations that occur in the natural environment such as a fear of heights or storms.

If clozapine (Clozaril) therapy is being considered, the nurse should evaluate which laboratory test to establish a baseline for comparison in order to recognize a potentially life-threatening side effect? A. White blood cell count B. Liver function studies C. Creatinine clearance D. Blood urea nitrogen

ANS: A The nurse should establish a baseline white blood cell count to evaluate a potentially life-threatening side effect if clozapine (Clozaril) is being considering as a treatment option. Clozapine can have a serious side effect of agranulocytosis in which a potentially fatal drop in white blood cells can occur.

After taking chlorpromazine (Thorazine) for 1 month, a client presents to an emergency department (ED) with severe muscle rigidity, tachycardia, and a temperature of 105oF (40.5C). The nurse expects the physician to recognize which condition and implement which treatment? A. Neuroleptic malignant syndrome and treat by discontinuing Thorazine and administering dantrolene (Dantrium) B. Neuroleptic malignant syndrome and treat by increasing Thorazine dosage and administering an antianxiety medication C. Dystonia and treat by administering trihexyphenidyl (Artane) D. Dystonia and treat by administering bromocriptine (Parlodel)

ANS: A The nurse should expect that an ED physician would diagnose the client with neuroleptic malignant syndrome and treat the client by discontinuing chlorpromazine (Thorazine) and administering dantrolene (Dantrium). Neuroleptic malignant syndrome is a potentially fatal condition characterized by muscle rigidity, fever, altered consciousness, and autonomic instability. The use of typical antipsychotics is largely being replaced by atypical antipsychotics due to fewer side effects and lower risks.

A 16-year-old client diagnosed with schizophrenia experiences command hallucinations to harm others. The clients parents ask a nurse, Where do the voices come from? Which is the appropriate nursing reply? A. Your child has a chemical imbalance of the brain, which leads to altered thoughts. B. Your childs hallucinations are caused by medication interactions. C. Your child has too little serotonin in the brain, causing delusions and hallucinations. D. Your childs abnormal hormonal changes have precipitated auditory hallucinations.

ANS: A The nurse should explain that a chemical imbalance of the brain leads to altered thought processes. Hallucinations, or false sensory perceptions, may occur in all five senses. The client who hears voices is experiencing an auditory hallucination.

A psychiatrist prescribes a monoamine oxidase inhibitor (MAOI) for a client. Which foods should the nurse teach the client to avoid? A. Pepperoni pizza and red wine B. Bagels with cream cheese and tea C. Apple pie and coffee D. Potato chips and diet cola

ANS: A The nurse should instruct the client to avoid pepperoni pizza and red wine. Foods with high tyramine content can induce hypertensive crisis within 2 hours of ingestion. Symptoms of hypertensive crisis include severe occipital and/or temporal pounding headaches with occasional photophobia, sensations of choking, palpitations, and a feeling of "dread."

A client diagnosed with schizophrenia is prescribed clozapine (Clozaril). Which client symptoms related to the side effects of this medication should prompt a nurse to intervene immediately? A. Sore throat, fever, and malaise B. Akathisia and hypersalivation C. Akinesia and insomnia D. Dry mouth and urinary retention

ANS: A The nurse should intervene immediately if the client experiences a sore throat, fever, and malaise when taking the atypical antipsychotic drug clozapine (Clozaril). Clozapine can have a serious side effect of agranulocytosis, in which a potentially fatal drop in white blood cells can occur. Symptoms of infectious processes would alert the nurse to this potential.

A client is prescribed alprazolam (Xanax) for acute anxiety. What client history should cause a nurse to question this order? A. History of alcohol dependence B. History of personality disorder C. History of schizophrenia D. History of hypertension

ANS: A The nurse should question a prescription of alprazolam (Xanax) for acute anxiety if the client has a history of alcohol dependence. Alprazolam is a benzodiazepine used in the treatment of anxiety and has an increased risk for physiological dependence and tolerance. A client with a history of substance abuse may be more likely to abuse other addictive substances and/or combine this drug with alcohol.

Which statement should indicate to a nurse that an individual is experiencing a delusion? A. "There's an alien growing in my liver." B. "I see my dead husband everywhere I go." C. "The IRS may audit my taxes." D. "I'm not going to eat my food. It smells like brimstone."

ANS: A The nurse should recognize that a client who claims that an alien is inside his or her body is experiencing a delusion. Delusions are false personal beliefs that are inconsistent with the person's intelligence or cultural background.

A client is taking chlordiazepoxide (Librium) for generalized anxiety disorder symptoms. In which situation should a nurse recognize that this client is at greatest risk for drug overdose? A. When the client has a knowledge deficit related to the effects of the drug B. When the client combines the drug with alcohol C. When the client takes the drug on an empty stomach D. When the client fails to follow dietary restrictions

ANS: B Both Librium and alcohol are central nervous system depressants. In combination, these drugs have an additive effect and can suppress the respiratory system leading to respiratory arrest and death.

A nursing instructor is teaching students about the differences between the symptoms of anorexia nervosa and the symptoms of bulimia nervosa. Which student statement indicates that learning has occurred? A. "Clients diagnosed with anorexia nervosa experience extreme nutritional deficits, whereas clients diagnosed with bulimia nervosa do not." B. "Clients diagnosed with bulimia nervosa experience amenorrhea, whereas clients diagnosed with anorexia nervosa do not." C. "Clients diagnosed with bulimia nervosa experience hypotension, edema, and lanugo, whereas clients diagnosed with anorexia nervosa do not." D. "Clients diagnosed with anorexia nervosa have eroded tooth enamel, whereas clients diagnosed with bulimia nervosa do not."

ANS: A The nursing student statement that clients diagnosed with anorexia nervosa experience nutritional deficits, whereas clients diagnosed with bulimia nervosa do not, indicates that learning has occurred. Anorexia is characterized by low caloric and nutritional intake. Bulimia is characterized by episodic, rapid indigestion of large quantities of food followed by purging.

A nursing instructor is teaching about the medications used to treat panic disorder. Which student statement indicates that learning has occurred? A. "Clonazepam (Klonopin) is particularly effective in the treatment of panic disorder." B. "Clozapine (Clozaril) is used off-label in long-term treatment of panic disorder." C. "Doxepin (Sinequan) can be used in low doses to relieve symptoms of panic attacks." D. "Buspirone (BuSpar) is used for its immediate effect to lower anxiety during panic attacks."

ANS: A The student indicates learning has occurred when he or she states that clonazepam is a particularly effective treatment for panic disorder. Clonazepam is a type of benzodiazepine that can be abused and lead to physical dependence and tolerance. It can be used on an as-needed basis to reduce anxiety and its related symptoms.

A nurse should identify topiramate (Topamax) as the drug of choice for which of the following conditions? (Select all that apply.) A. Binge eating with obesity B. Bingeing and purging with a diagnosis of bulimia nervosa C. Weight loss with a diagnosis of anorexia nervosa D. Amenorrhea with a diagnosis of anorexia nervosa E. Emaciation with a diagnosis of bulimia nervosa

ANS: A, B The nurse should identify that topiramate (Topamax) is the drug of choice when treating binge eating with obesity and bingeing and purging with a diagnosis of bulimia nervosa. Topiramate (Topamax) is a novel anticonvulsant used in the long-term treatment of binge-eating disorder with obesity. The use of Topamax results in a significant decline in mean weekly binge frequency and significant reduction in body weight. With the use of this medication, episodes of bingeing and purging were decreased in clients diagnosed with bulimia nervosa.

Laboratory results reveal decreased levels of prolactin in a client diagnosed with schizophrenia. When assessing the client, which symptoms should a nurse expect to observe?(Select all that apply.) A. Apathy B. Social withdrawal C. Anhedonia D. Auditory hallucinations E. Delusions

ANS: A, B, C The nurse should expect that a client with decreased levels of prolactin would experience apathy, social withdrawal, and anhedonia. Decreased levels of prolactin can cause depression which would result in the above symptoms.

A client who has been diagnosed with a phobic disorder asks the nurse if there are any medications that would be beneficial in treating phobic disorders. Which of the following would be accurate responses by the nurse? Select all that apply. A. Some antianxiety agents have been successful in treating social phobias. B. Some antidepressant agents have been successful in diminishing symptoms of agoraphobia and social anxiety disorder (social phobia). C. Specific phobias are generally not treated with medication unless accompanied by panic attacks. D. Beta-blockers have been used successfully to treat phobic responses to public performance.

ANS: A, B, C, D All of the listed pharmacological treatments are evidence-based treatments for phobic disorders.

A client is prescribed phenelzine (Nardil). Which of the following client statements should indicate to a nurse that discharge teaching about this medication has been successful? (Select all that apply.) A. "I'll have to let my surgeon know about this medication before I have my cholecystectomy." B. "Guess I will have to give up my glass of red wine with dinner." C. "I'll have to be very careful about reading food and medication labels." D. "I'm going to miss my caffeinated coffee in the morning." E. "I'll be sure not to stop this medication abruptly."

ANS: A, B, C, E The nurse should evaluate that teaching has been successful when the client states that phenelzine (Nardil) should not be taken in conjunction with the use of alcohol or foods high in tyramine and should not be stopped abruptly. Phenelzine is a monoamine oxidase inhibitor (MAOI) that can have negative interactions with other medications. The client needs to tell other physicians about taking MAOIs due to the risk of drug interactions.

Which of the following components should a nurse recognize as an integral part of a rehabilitative program when planning care for clients diagnosed with schizophrenia? (Select all that apply.) A. Group therapy B. Medication management C. Deterrent therapy D. Supportive family therapy E. Social skills training

ANS: A, B, D, E The nurse should recognize that group therapy, medication management, supportive family therapy, and social skills training all play an integral part in rehabilitative programs for clients diagnosed with schizophrenia. Schizophrenia results from various combinations of genetic predispositions, biochemical dysfunctions, physiological factors, and psychological stress. Effective treatment requires a comprehensive, multidisciplinary effort.

A 20-year-old female has a diagnosis of premenstrual dysphoric disorder. Which of the following should a nurse identify as consistent with this diagnosis? Select all that apply. A. Symptoms are causing significant interference with work, school, and social relationships. B. Patient-rated mood is 2/10 for the past 6 months C. Mood swings occur the week before onset of menses D. Patient reports subjective difficulty concentrating E. Patient manifests pressured speech when communicating

ANS: A, C, D Diagnostic criteria for a premenstrual dysphoric disorder include that symptoms must be associated with significant distress, occur in the week before onset of menses, and improve or disappear in the week post-menses

A paranoid client presents with bizarre behaviors, neologisms, and thought insertion. Which nursing action should be prioritized to maintain this clients safety? A. Assess for medication noncompliance B. Note escalating behaviors and intervene immediately C. Interpret attempts at communication D. Assess triggers for bizarre, inappropriate behaviors

ANS: B The nurse should note escalating behaviors and intervene immediately to maintain this clients safety. Early intervention may prevent an aggressive response and keep the client and others safe.

A nurse has been caring for a client diagnosed with generalized anxiety disorder (GAD). Which of the following nursing interventions would address this client's symptoms? (Select all that apply.) A. Encourage the client to recognize the signs of escalating anxiety. B. Encourage the client to avoid any situation that causes stress. C. Encourage the client to employ newly learned relaxation techniques. D. Encourage the client to cognitively reframe thoughts about situations that generate anxiety. E. Encourage the client to avoid caffeinated products.

ANS: A, C, D, E Nursing interventions that address GAD symptoms should include encouraging the client to recognize signs of escalating anxiety, to employ relaxation techniques, to cognitively reframe thoughts about anxiety-provoking situations, and to avoid caffeinated products. Avoiding situations that cause stress is not an appropriate intervention because avoidance does not help the client overcome anxiety. Stress is a component of life and is not easily evaded.

A nurse is administering risperidone (Risperdal) to a client diagnosed with schizophrenia. The therapeutic effect of this medication would most effectively address which of the following symptoms? (Select all that apply.) A. Somatic delusions B. Social isolation C. Gustatory hallucinations D. Flat affect E. Clang associations

ANS: A, C, E The nurse should expect that risperidone (Risperdal) would be effective treatment for somatic delusions, gustatory hallucinations, and clang associations. Risperidone is an atypical antipsychotic that has been effective in the treatment of the positive symptoms of schizophrenia and in maintenance therapy to prevent exacerbation of schizophrenic symptoms.

A nursing home resident has a diagnosis of dysthymic disorder. When planning care for this client, which of the following symptoms should a nurse expect the client to exhibit? (Select all that apply.) A. Sad mood on most days B. Mood rating of 2/10 for the past 6 months C. Labile mood D. Sad mood for the past 3 years after spouse's death E. Pressured speech when communicating

ANS: A, D The nurse should anticipate that a client with a diagnosis of dysthymic disorder would experience a sad mood on most days for more than 2 years. The essential feature of dysthymia is a chronically depressed mood which can have an early or late onset.

A college student has been diagnosed with generalized anxiety disorder (GAD). Which of the following symptoms should a campus nurse expect this client to exhibit? (Select all that apply.) A. Fatigue B. Anorexia C. Hyperventilation D. Insomnia E. Irritability

ANS: A, D, E The nurse should expect that a client diagnosed with GAD would experience fatigue, insomnia, and irritability. GAD is characterized by chronic, unrealistic, and excessive anxiety and worry.

An isolative client was admitted 4 days ago with a diagnosis of major depressive disorder. Which nursing statement would best motivate this client to attend a therapeutic group being held in the milieu? A. "We'll go to the day room when you are ready for group." B. "I'll walk with you to the day room. Group is about to start." C. "It must be difficult for you to attend group when you feel so bad." D. "Let me tell you about the benefits of attending this group."

ANS: B A client diagnosed with major depressive disorder exhibits little to no motivation and must be firmly directed by staff to participate in therapy. It is difficult for a severely depressed client to make decisions, and this function must be temporarily assumed by the staff.

Which nursing intervention is appropriate when caring for clients diagnosed with either anorexia nervosa or bulimia nervosa? A. Provide privacy during meals. B. Remain with the client for at least 1 hour after the meal. C. Encourage the client to keep a journal to document types of food consumed. D. Restrict client privileges when provided food is not completely consumed.

ANS: B A nurse should remain with clients diagnosed with either anorexia nervosa or bulimia nervosa for at least 1 hour after meals. This allows the nurse to monitor for food discarding (anorexia nervosa) and/or self-induced vomiting (bulimia nervosa).

A client is diagnosed with major depressive disorder. Which nursing diagnosis should a nurse assign to this client to address a behavioral symptom of this disorder? A. Altered communication R/T feelings of worthlessness AEB anhedonia B. Social isolation R/T poor self-esteem AEB secluding self in room C. Altered thought processes R/T hopelessness AEB persecutory delusions D. Altered nutrition: less than body requirements R/T high anxiety AEB anorexia

ANS: B A nursing diagnosis of social isolation R/T poor self-esteem AEB secluding self in room addresses a behavioral symptom of major depressive disorder. Other behavioral symptoms include psychomotor retardation, virtually nonexistent communication, maintaining a fetal position, and no personal hygiene and/or grooming.

A client presents in the emergency department with complaints of overwhelming anxiety. Which of the following is a priority for the nurse to assess? A. Risk for suicide B. Cardiac status C. Current stressors D. Substance use history

ANS: B Although all of the listed aspects of assessment are important, the priority is to evaluate cardiac status since a person having an MI, CHF, or mitral valve prolapse can present with symptoms of anxiety.

A client diagnosed with obsessive-compulsive disorder is admitted to a psychiatric unit. The client has an elaborate routine for toileting activities. Which would be an appropriate initial client outcome during the first week of hospitalization? A. The client will refrain from ritualistic behaviors during daylight hours. B. The client will wake early enough to complete rituals prior to breakfast. C. The client will participate in three unit activities by day 3. D. The client will substitute a productive activity for rituals by day 1.

ANS: B An appropriate initial client outcome is for the client to wake early enough to complete rituals prior to breakfast. The nurse should also provide a structured schedule of activities and later in treatment begin to gradually limit the time allowed for rituals.

A client who is 5 foot 6 inches tall and weighs 98 pounds is admitted with a medical diagnosis of anorexia nervosa. Which nursing diagnosis would take priority at this time? A. Ineffective coping R/T food obsession B. Altered nutrition: less than body requirements R/T inadequate food intake C. Risk for injury R/T suicidal tendencies D. Altered body image R/T perceived obesity

ANS: B Based on Maslow's hierarchy, the priority nursing diagnosis for this client must address physical needs prior to emotional considerations. This client must be immediately physically stabilized due to the life-threatening nature of his or her nutritional status.

A client who has been taking buspirone (BuSpar) as prescribed for 2 days is close to discharge. Which statement indicates to the nurse that the client has an understanding of important discharge teaching? A. "I cannot drink any alcohol with this medication." B. "It is going to take 2 to 3 weeks in order for me to begin to feel better." C. "This drug causes physical dependence and I need to strictly follow doctor's orders." D. "I can't take this medication with food. It needs to be taken on an empty stomach."

ANS: B Buspar takes at least 2 to 3 weeks to be effective in controlling symptoms of depression. This is important to teach clients in order to prevent potential noncompliance due to the perception that the medication is ineffective.

A newly admitted client diagnosed with major depressive disorder states, "I have never considered suicide." Later the client confides to the nurse about plans to end it all by medication overdose. What is the most helpful nursing reply? A. "I'm glad you shared this. There is nothing to worry about. We will handle it together." B. "Bringing this up is a very positive action on your part." C. "We need to talk about the things you have to live for." D. "I think you should consider all your options prior to taking this action."

ANS: B By admitting to the staff a suicide plan, this client has taken responsibility for possible personal actions and expresses trust in the nurse. Therefore, the client may be receptive to continuing a safety plan. Recognition of this achievement reinforces this adaptive behavior.

A nurse working with a client diagnosed with bulimia nervosa asks the client to recall a time in life when food could be consumed without purging. Which is the purpose of this nursing intervention? A. To gain additional information about the progression of the disease process B. To emphasize that the client is capable of consuming food without purging C. To incorporate specific foods into the meal plan to reflect pleasant memories D. To assist the client to become more compliant with the treatment plan

ANS: B By asking the client to recall a time in life when food could be consumed without purging, the nurse is assessing previously successful coping strategies. This information can be used by the client to modify maladaptive behaviors in the present and future.

A client who is diagnosed with major depressive disorder asks the nurse what causes depression. Which of these is the most accurate response? A. Depression is caused by a deficiency in neurotransmitters, including serotonin and norepinephrine. B. The exact cause of depressive disorders is unknown. A number of things, including genetic, biochemical, and environmental influences, likely play a role. C. Depression is a learned state of helplessness cause by ineffective parenting. D. Depression is caused by intrapersonal conflict between the id and the ego.

ANS: B Depression is likely an illness that has varied and multiple causative factors, but at present the exact cause of depressive disorders is not entirely understood.

A client admitted to the psychiatric unit following a suicide attempt is diagnosed with major depressive disorder. Which behavioral symptoms should the nurse expect to assess? A. Anxiety and unconscious anger B. Lack of attention to grooming and hygiene C. Guilt and indecisiveness D. Expressions of poor self-esteem

ANS: B Lack of attention to grooming and hygiene is the only behavioral symptom presented. Depressed clients do not care enough about themselves to participate in grooming and hygiene.

A nurse is implementing a one-on-one suicide observation level with a client diagnosed with major depressive disorder. The client states, "I'm feeling a lot better so you can stop watching me. I have taken up too much of your time already." Which is the best nursing reply? A. "I really appreciate your concern but I have been ordered to continue to watch you." B. "Because we are concerned about your safety, we will continue to observe you." C. "I am glad you are feeling better. The treatment team will consider your request." D. "I will forward you request to your psychiatrist because it is his decision."

ANS: B Often suicidal clients resist personal monitoring which impedes the implementation of a suicide plan. A nurse should continually observe a client when risk for suicide is suspected.

Which nursing diagnosis would best describe the problems evidenced by the following client symptoms: avoidance, poor concentration, nightmares, hypervigilance, exaggerated startle response, detachment, emotional numbing, and flashbacks? A. Ineffective coping B. Post-trauma syndrome C. Complicated grieving D. Panic anxiety

ANS: B Post-trauma syndrome is defined as a sustained maladaptive response to a traumatic, overwhelming event. This nursing diagnosis addresses the problems experienced by clients diagnosed with post-traumatic stress disorder.

A nursing student questions an instructor regarding the order for fluvoxamine (Luvox) 300 mg daily for a client diagnosed with obsessive-compulsive disorder (OCD). Which instructor reply is most accurate? A. "High doses of tricyclic medications will be required for effective treatment of OCD." B. "Selective serotonin reuptake inhibitor (SSRI) doses, in excess of what is effective for treating depression, may be required for OCD." C. "The dose of Luvox is low due to the side effect of daytime drowsiness and nighttime insomnia." D. "The dosage of Luvox is outside the therapeutic range and needs to be questioned."

ANS: B The most accurate instructor response is that SSRI doses, in excess of what is effective for treating depression, may be required in the treatment of OCD. SSRIs have been approved by the U.S. Food and Drug Administration for the treatment of OCD. Common side effects include headache, sleep disturbances, and restlessness.

A family member is seeking advice about an elderly parent who seems to worry unnecessarily about everything. The family member states, "Should I seek psychiatric help for my mother?" Which is an appropriate nursing reply? A. "My mother also worries unnecessarily. I think it is part of the aging process." B. "Anxiety is considered abnormal when it is out of proportion to the stimulus causing it and when it impairs functioning." C. "From what you have told me, you should get her to a psychiatrist as soon as possible." D. "Anxiety is a complex phenomenon and is effectively treated only with psychotropic medications."

ANS: B The most appropriate reply by the nurse is to explain to the family member that anxiety is considered abnormal when it is out of proportion and impairs functioning. Anxiety is a normal reaction to a realistic danger or threat to biological integrity or self-concept.

A client on an inpatient unit is diagnosed with bipolar disorder: manic episode. During a discussion in the dayroom about weekend activities, the client raises his voice, becomes irritable, and insists that plans change. What should be the nurse's initial intervention?

Assist the client to move to a calmer location.

A newly admitted client is diagnosed with major depressive disorder with suicidal ideations. Which would be the priority nursing intervention for this client? A. Teach about the effective of suicide on family dynamics. B. Carefully and unobtrusively observe based on assessed data, at varied intervals around the clock. C. Encourage the client to spend a portion of each day interacting within the milieu. D. Set realistic achievable goals to increase self esteem.

ANS: B The most effective way to interrupt a suicide attempt is to carefully, unobtrusively observe based on assessed data at varied intervals around the clock. If a nurse observes behavior that indicates self-harm, the nurse can intervene to stop the behavior and keep the client safe.

Which nursing behavior will enhance the establishment of a trusting relationship with a client diagnosed with schizophrenia? A. Establishing personal contact with family members. B. Being reliable, honest, and consistent during interactions. C. Sharing limited personal information. D. Sitting close to the client to establish rapport.

ANS: B The nurse can enhance the establishment of a trusting relationship with a client diagnosed with schizophrenia by being reliable, honest, and consistent during interactions. The nurse should also convey acceptance of the clients needs and maintain a calm attitude when dealing with agitated behavior.

During an admission assessment, a nurse asks a client diagnosed with schizophrenia, "Have you ever felt that certain objects or persons have control over your behavior?" The nurse is assessing for which type of thought disruption? A. Delusions of persecution B. Delusions of influence C. Delusions of reference D. Delusions of grandeur

ANS: B The nurse is assessing the client for delusions of influence when asking if the client has ever felt that objects or persons have control of the client's behavior. Delusions of control or influence are manifested when the client believes that his or her behavior is being influenced. An example would be if a client believes that a hearing aid receives transmissions that control personal thoughts and behaviors.

A client diagnosed with schizophrenia tells a nurse, The Shopatouliens took my shoes out of my room last night. Which is an appropriate charting entry to describe this clients statement? A. The client is experiencing command hallucinations. B. The client is expressing a neologism. C. The client is experiencing a paranoia. D. The client is verbalizing a word salad.

ANS: B The nurse should describe the clients statement as experiencing a neologism. A neologism is when a client invents a new word that is meaningless to others but may have symbolic meaning to the client. Word salad refers to a group of words that are put together randomly.

What is the rationale for a nurse to perform a full physical health assessment on a client admitted with a diagnosis of major depressive disorder? A. The attention during the assessment is beneficial in decreasing social isolation. B. Depression can generate somatic symptoms that can mask actual physical disorders. C. Physical health complications are likely to arise from antidepressant therapy. D. Depressed clients avoid addressing physical health and ignore medical problems.

ANS: B The nurse should determine that a client with a diagnosis of major depressive disorder needs a full physical health assessment because depression can generate somatic symptoms that can mask actual physical disorders. Somatization is the process by which psychological needs are expressed in the form of physical symptoms.

The family of a client diagnosed with anorexia nervosa becomes defensive when the treatment team calls for a family meeting. Which is the appropriate nursing reply? A. "Tell me why this family meeting is causing you to be defensive. All clients are required to participate in two family sessions." B. "Family intervention and support are important in your child's recovery." C. "Family dynamics are not linked to eating disorders. The meeting is to provide your child with family support." D. "Clients diagnosed with anorexia nervosa are part of the family system, and any alteration in family processes needs to be addressed."

ANS: B The nurse should educate the family on the correlation between certain familial patterns and anorexia nervosa. Families engaging in conflict avoidance and struggling with issues of power and control may contribute to the development of this disorder.

A client refuses to go on a cruise to the Bahamas with his spouse due to fearing that the cruise ship will sink and all will drown. Using a cognitive theory perspective, how should a nurse explain to the spouse the etiology of this fear? A. "Your spouse may be unable to resolve internal conflicts which result in projected anxiety." B. "Your spouse may be experiencing a distorted and unrealistic appraisal of the situation." C. "Your spouse may have a genetic predisposition to overreacting to potential danger." D. "Your spouse may have high levels of brain chemicals that may distort thinking."

ANS: B The nurse should explain that from a cognitive perspective the client is experiencing a distorted and unrealistic appraisal of the situation. From a cognitive perspective, fear is described as the result of faulty cognitions.

A client who has been taking fluvoxamine (Luvox) without significant improvement asks a nurse, "I heard about something called a monoamine oxidase inhibitor (MAOI). Can't my doctor add that to my medications?" Which is an appropriate nursing reply? A. "This combination of drugs can lead to delirium tremens." B. "A combination of an MAOI and Luvox can lead to a life-threatening hypertensive crisis." C. "That's a good idea. There have been good results with the combination of these two drugs." D. "The only disadvantage would be the exorbitant cost of the MAOI."

ANS: B The nurse should explain to the client that combining an MAOI and Luvox can lead to a life-threatening hypertensive crisis. Symptoms of hypertensive crisis include severe occipital and/or temporal pounding headaches with occasional photophobia, sensations of choking, palpitations, and a feeling of "dread."

A client has been recently admitted to an inpatient psychiatric unit. Which intervention should the nurse plan to use to reduce the clients focus on delusional thinking? A. Present evidence that supports the reality of the situation B. Focus on feelings suggested by the delusion C. Address the delusion with logical explanations D. Explore reasons why the client has the delusion

ANS: B The nurse should focus on the clients feelings rather than attempt to change the clients delusional thinking by the use of evidence or logical explanations. Delusional thinking is usually fixed, and clients will continue to have the belief in spite of obvious proof that the belief is false or irrational.

A nurse is caring for a client who is experiencing a flat affect, paranoid delusions, anhedonia, anergia, neologisms, and echolalia. Which statement correctly differentiates the client's positive and negative symptoms of schizophrenia? A. Paranoid delusions, anhedonia, and anergia are positive symptoms of schizophrenia. B. Paranoid delusions, neologisms, and echolalia are positive symptoms of schizophrenia. C. Paranoid delusions, anergia, and echolalia are negative symptoms of schizophrenia. D. Paranoid delusions, flat affect, and anhedonia are negative symptoms of schizophrenia.

ANS: B The nurse should recognize that positive symptoms of schizophrenia include paranoid delusions, neologisms, and echolalia. The negative symptoms of schizophrenia include flat affect, anhedonia, and anergia. Positive symptoms reflect an excess or distortion of normal functions. Negative symptoms reflect a decrease or loss of normal functions.

A college student has quit attending classes, isolates self because of hearing voices, and yells accusations at fellow students. Based on this information, which nursing diagnosis should the nurse prioritize? A. Altered thought processes R/T hearing voices AEB increased anxiety B. Risk for other-directed violence R/T yelling accusations C. Social isolation R/T paranoia AEB absence from classes D. Risk for self-directed violence R/T depressed mood

ANS: B The nursing diagnosis that must be prioritized in this situation is risk for other-directed violence R/T yelling accusations. Hearing voices and yelling accusations indicate a potential for violence, and this potential safety issue should be prioritized.

A nursing instructor is teaching about specific phobias. Which student statement should indicate that learning has occurred? A. "These clients do not recognize that their fear is excessive and rarely seek treatment." B. "These clients have overwhelming symptoms of panic when exposed to the phobic stimulus." C. "These clients experience symptoms that mirror a cerebrovascular accident (CVA)." D. "These clients experience the symptoms of tachycardia, dysphagia, and diaphoresis."

ANS: B The nursing instructor should evaluate that learning has occurred when the student knows that clients experiencing phobias have a panic level of fear that is overwhelming and unreasonable. Phobia is fear cued by a specific object or situation in which exposure to the stimuli produces an immediate anxiety response.

A client has the following symptoms: preoccupation with imagined defect, verbalizations that are out of proportion to actual physical abnormalities, and numerous visits to plastic surgeons to seek relief. Which nursing diagnosis would best describe the problems evidenced by these symptoms? A. Ineffective coping B. Disturbed body image C. Complicated grieving D. Panic anxiety

ANS: B The symptoms presented describe the DSM-5 diagnosis of body dysmorphic disorder, and the related nursing diagnosis is disturbed body image.

A client is admitted with a diagnosis of depression NOS (not otherwise specified). Which client statement would describe a somatic symptom that can occur with this diagnosis? A. "I am extremely sad, but I don't know why." B. "Sometimes I just don't want to eat because I ache all over." C. "I feel like I can't ever make the right decision." D. "I can't seem to leave the house without someone with me."

ANS: B When a client diagnosed with depression expresses physical complaints, the client is experiencing somatic symptoms. Somatic symptoms occur with depression because of a general slowdown of the entire body reflected in sluggish digestion, constipation, impotence, anorexia, difficulty falling asleep, and a wide variety of other symptoms.

A nurse is discussing treatment options with a client whose life has been negatively impacted by claustrophobia. The nurse would expect which of the following behavioral therapies to be most commonly used in the treatment of phobias? (Select all that apply.) A. Benzodiazepine therapy B. Systematic desensitization C. Imploding (flooding) D. Assertiveness training E. Aversion therapy

ANS: B, C The nurse should explain to the client that systematic desensitization and imploding are the most commonly used behavioral therapies in the treatment of phobias. Systematic desensitization involves the gradual exposure of the client to anxiety-provoking stimuli. Imploding is the intervention used in which the client is exposed to extremely frightening stimuli for prolonged periods of time.

A client who is a veteran of the Gulf War is being assessed by a nurse for post-traumatic stress disorder (PTSD). Which of the following client symptoms would support this diagnosis? (Select all that apply.) A. The client has experienced symptoms of the disorder for 2 weeks. B. The client fears a physical integrity threat to self. C. The client feels detached and estranged from others. D. The client experiences fear and helplessness. E. The client is lethargic and somnolent.

ANS: B, C, D Clients diagnosed with PTSD can experience the following symptoms: fear of a physical integrity threat to self, detachment and estrangement from others, and intense fear and helplessness. Characteristic symptoms of PTSD include re-living the traumatic event, a sustained high level of arousal, and a general numbing of responsiveness.

An individual experiences sadness and melancholia in September continuing through November. Which of the following factors should a nurse identify as most likely to contribute to the etiology of these symptoms? (Select all that apply.) A. Gender differences in social opportunities that occur with age B. Drastic temperature and barometric pressure changes C. Increased levels of melatonin D. Variations in serotonergic functioning E. Inaccessibility of resources for dealing with life stressors

ANS: B, C, D The nurse should identify drastic temperature and barometric pressure changes, increased levels of melatonin, and/or variations in serotonergic functioning as contributing to the etiology of the client's symptoms. A number of studies have examined seasonal patterns associated with mood disorders and have revealed two prevalent periods of seasonal involvement: spring (March, April, May) and fall (September, October, November).

A nurse admits an older client who is experiencing memory loss, confused thinking, and apathy. A psychiatrist suspects depression. What is the rationale for performing a mini-mental status exam? A. To rule out bipolar disorder B. To rule out schizophrenia C. To rule out senile dementia D. To rule out a personality disorder

ANS: C A mini-mental status exam should be performed to rule out senile dementia. The elderly are often misdiagnosed with senile dementia when depression is their actual diagnosis. Memory loss, confused thinking, or apathy symptomatic of dementia actually may be the result of depression.

A 75-year-old client diagnosed with a long history of depression is currently on doxepin (Sinequan) 100 mg daily. The client takes a daily diuretic for hypertension and is recovering from the flu. Which nursing diagnosis should the nurse assign highest priority? A. Risk for ineffective thermoregulation R/T anhidrosis B. Risk for constipation R/T excessive fluid loss C. Risk for injury R/T orthostatic hypotension D. Risk for infection R/T suppressed white blood cell count

ANS: C A side effect of Sinequan is orthostatic hypotension. Dehydration due to fluid loss from a combination of diuretic medication and flu symptoms can also contribute to this problem, putting this client at risk for injury R/T orthostatic hypotension.

A newly admitted client has taken thioridazine (Mellaril) for 2 years, with good symptom control. Symptoms exhibited on admission included paranoia and hallucinations. The nurse should recognize which potential cause for the return of these symptoms? A. The client has developed tolerance to the antipsychotic medication. B. The client has not taken the medication with food. C. The client has not taken the medication as prescribed. D. The client has combined alcohol with the medication.

ANS: C Altered thinking can affect a clients insight into the necessity for taking antipsychotic medications consistently. When symptoms are no longer bothersome, clients may stop taking medications that cause disturbing side effects. Clients may miss the connection between taking the medications and an improved symptom profile.

How would a nurse differentiate a client diagnosed with a social phobia from a client diagnosed with a schizoid personality disorder (SPD)? A. Clients diagnosed with social phobia can manage anxiety without medications, whereas clients diagnosed with SPD can manage anxiety only with medications. B. Clients diagnosed with SPD are distressed by the symptoms experienced in social settings, whereas clients diagnosed with social phobia are not. C. Clients diagnosed with social phobia avoid interactions only in social settings, whereas clients diagnosed with SPD avoid interactions in all areas of life. D. Clients diagnosed with SPD avoid interactions only in social settings, whereas clients diagnosed with social phobias tend to avoid interactions in all areas of life.

ANS: C Clients diagnosed with social phobia avoid interactions only in social settings, whereas clients diagnosed with SPD avoid interactions in all areas of life. Social phobia is an excessive fear of situations in which a person might do something embarrassing or be evaluated negatively by others.

The nurse is providing counseling to clients diagnosed with major depressive disorder. The nurse chooses to help the clients alter their mood by learning how to change the way they think. The nurse is functioning under which theoretical framework? A. Psychoanalytic theory B. Interpersonal theory C. Cognitive theory D. Behavioral theory

ANS: C Cognitive theory suggests that depression is a product of negative thinking. Helping the individual change the way they think is believed to have a positive impact on mood and self-esteem.

When a community health nurse arrives at the home of a client diagnosed with bulimia nervosa, the nurse finds the client on the floor unconscious. The client has a history of using laxatives for purging. To what would the nurse attribute this client's symptoms? A. Increased creatinine and blood urea nitrogen (BUN) levels B. Abnormal electroencephalogram (EEG) C. Metabolic acidosis D. Metabolic alkalosis

ANS: C Excessive vomiting and laxative or diuretic abuse may lead to problems with dehydration and electrolyte imbalance. The nurse should attribute this client's fainting to the loss of alkaline stool due to laxative abuse which would lead to a relative metabolic acidotic condition.

A client states, I hear voices that tell me that I am evil. Which outcome related to these symptoms should the nurse expect this client to accomplish by discharge? A. The client will verbalize the reason the voices make derogatory statements. B. The client will not hear auditory hallucinations. C. The client will identify events that increase anxiety and illicit hallucinations. D. The client will positively integrate the voices into the clients personality structure.

ANS: C It is unrealistic to expect the client to completely stop hearing voices. Even when compliant with antipsychotic medications, clients may still hear voices. It would be realistic to expect the client to associate stressful events with an increase in auditory hallucinations. By this recognition the client can anticipate symptoms and initiate appropriate coping skills.

Warrens college roommate actively resists going out with friends whenever they invite him. He says he cant stand to be around other people and confides to Warren They wouldnt like me anyway. Which disorder is Warrens roommate likely suffering from? A. Agoraphobia B. Mysophobia C. Social anxiety disorder (social phobia) D. Panic disorder

ANS: C Social anxiety disorder is an excessive fear of social situations R/T fear that one might do something embarrassing or be evaluated negatively by others.

How should a nurse best describe the major maladaptive client response to panic disorder? A. Clients overuse medical care due to physical symptoms. B. Clients use illegal drugs to ease symptoms. C. Clients perceive having no control over life situations. D. Clients develop compulsions to deal with anxiety.

ANS: C The major maladaptive client response to panic disorder is the perception of having no control over life situations which leads to nonparticipation in decision making and doubts regarding role performance.

A client diagnosed with schizophrenia states, Cant you hear him? Its the devil. Hes telling me Im going to hell. Which is the most appropriate nursing reply? A. Did you take your medicine this morning? B. You are not going to hell. You are a good person. C. Im sure the voices sound scary. I dont hear any voices speaking. D. The devil only talks to people who are receptive to his influence.

ANS: C The most appropriate reply by the nurse is to reassure the client with an accepting attitude while not reinforcing the hallucination.

Parents ask a nurse how they should reply when their child, diagnosed with paranoid schizophrenia, tells them that voices command him to harm others. Which is the appropriate nursing reply? A. "Tell him to stop discussing the voices." B. "Ignore what he is saying, while attempting to discover the underlying cause." C. "Focus on the feelings generated by the hallucinations and present reality." D. "Present objective evidence that the voices are not real."

ANS: C The most appropriate response by the nurse is to instruct the parents to focus on the feelings generated by the hallucinations and present reality. The parents should maintain an attitude of acceptance to encourage communication but should not reinforce the hallucinations by exploring details of content. It is inappropriate to present logical arguments to persuade the client to accept the hallucinations as not real.

An elderly client diagnosed with schizophrenia takes an antipsychotic and a beta-adrenergic blocking agent (propranolol) for hypertension. Understanding the combined side effects of these drugs, the nurse would most appropriately make which statement? A. Make sure you concentrate on taking slow, deep, cleansing breaths. B. Watch your diet and try to engage in some regular physical activity. C. Rise slowly when you change position from lying to sitting or sitting to standing. D. Wear sunscreen and try to avoid midday sun exposure.

ANS: C The most appropriate statement by the nurse is to instruct the client to rise slowly when changing positions. Antipsychotic medications and beta blockers cause a decrease in blood pressure. When given in combination, this side effect places the client at risk for developing orthostatic hypotension.

A client is experiencing a severe panic attack. Which nursing intervention would meet this client's immediate need? A. Teach deep breathing relaxation exercises B. Place the client in a Trendelenburg position C. Stay with the client and offer reassurance of safety D. Administer the ordered prn buspirone (BuSpar)

ANS: C The nurse can meet this client's immediate need by staying with the client and offering reassurance of safety and security. The client may fear for his or her life and the presence of a trusted individual provides assurance of personal safety.

Family dynamics are thought to be a major influence in the development of anorexia nervosa. Which information related to a client's home environment should a nurse associate with the development of this disorder? A. The home environment maintains loose personal boundaries. B. The home environment places an overemphasis on food. C. The home environment is overprotective and demands perfection. D. The home environment condones corporal punishment.

ANS: C The nurse should assess that a home environment that is overprotective and demands perfection may be a major influence in the development of anorexia nervosa. In adolescence, distorted eating patterns may represent a rebellion against controlling and demanding parents.

A depressed client reports to a nurse a history of divorce, job loss, family estrangement, and cocaine abuse. Which theoretical principle best explains the etiology of this client's depressive symptoms? A. According to psychoanalytic theory, depression is a result of anger turned inward. B. According to object-loss theory, depression is a result of abandonment. C. According to learning theory, depression is a result of repeated failures. D. According to cognitive theory, depression is a result of negative perceptions.

ANS: C The nurse should assess that this client's depressive symptoms may have resulted from repeated failures. This assessment was based on the principles of learning theory. Learning theory describes a model of "learned helplessness" in which multiple life failures cause the client to abandon future attempts to succeed.

A client diagnosed with schizophrenia states, My psychiatrist is out to get me. Im sad that the voice is telling me to stop him. What symptom is the client exhibiting, and what is the nurses legal responsibility related to this symptom? A. Magical thinking; administer an antipsychotic medication B. Persecutory delusions; orient the client to reality C. Command hallucinations; warn the psychiatrist D. Altered thought processes; call an emergency treatment team meeting

ANS: C The nurse should determine that the client is exhibiting command hallucinations. The nurses legal responsibility is to warn the psychiatrist of the potential for harm. A client who is demonstrating a risk for violence could potentially become physically, emotionally, and/or sexually harmful to others or to self.

A client diagnosed with schizophrenia is slow to respond and appears to be listening to unseen others. Which medication should a nurse expect a physician to order to address this type of symptom? A. Haloperidol (Haldol) to address the negative symptom B. Clonazepam (Klonopin) to address the positive symptom C. Risperidone (Risperdal) to address the positive symptom D. Clozapine (Clozaril) to address the negative symptom

ANS: C The nurse should expect the physician to order risperidone (Risperdal) to address the positive symptoms of schizophrenia. Risperidone (Risperdal) is an atypical antipsychotic used to reduce positive symptoms, including disturbances in content of thought (delusions), form of thought (neologisms), or sensory perception (hallucinations).

A client living on the beachfront seeks help with an extreme fear of crossing bridges which interferes with daily life. A psychiatric nurse practitioner decides to try systematic desensitization. Which explanation of this therapy should the nurse convey to the client? A. "Using your imagination, we will attempt to achieve a state of relaxation that you can replicate when faced with crossing a bridge." B. "Because anxiety and relaxation are mutually exclusive states, we can attempt to substitute a relaxation response for the anxiety response." C. "Through a series of increasingly anxiety-provoking steps, we will gradually increase your tolerance to anxiety." D. "In one intense session, you will be exposed to a maximum level of anxiety that you will learn to tolerate."

ANS: C The nurse should explain to the client that systematic desensitization exposes the client to a series of increasingly anxiety provoking steps that will gradually increase anxiety tolerance. Systematic desensitization was introduced by Joseph Wolpe in 1958 and is based on behavioral conditioning principles.

A client diagnosed with post-traumatic stress disorder is receiving paliperidone (Invega). Which symptoms should a nurse identify that warrant the need for this medication? A. Flat affect and anhedonia B. Persistent anorexia and 10 lb weight loss in 3 weeks C. Flashbacks of killing the enemy D. Distant and guarded relationships

ANS: C The nurse should identify that a client who has flashbacks of killing the enemy may need paliperidone (Invega). Paliperidone is an antipsychotic medication that can be used to treat the psychotic symptom of flashbacks.

Which treatment should a nurse identify as most appropriate for clients diagnosed with generalized anxiety disorder (GAD)? A. Long-term treatment with diazepam (Valium) B. Acute symptom control with citalopram (Celexa) C. Long-term treatment with buspirone (BuSpar) D. Acute symptom control with ziprasidone (Geodon)

ANS: C The nurse should identify that an appropriate treatment for clients diagnosed with GAD is long-term treatment with buspirone. Buspirone is an anxiolytic medication that is effective in 60% to 80% of clients with GAD. It takes 10 to 14 days for alleviation of symptoms but does not have the dependency concerns of other anxiolytics.

A high school senior is diagnosed with anorexia nervosa and is hospitalized for severe malnutrition. The treatment team is planning to use behavior modification. What rationale should a nurse identify as the reasoning behind this therapy choice? A. This therapy will increase the client's motivation to gain weight. B. This therapy will reward the client for perfectionist achievements. C. This therapy will provide the client with control over behavioral choices. D. This therapy will protect the client from parental overindulgence.

ANS: C The nurse should identify that behavior modification therapy will be used because it provides the client with control over behavioral choices. Clients diagnosed with anorexia nervosa are often allowed to contract privileges based on weight gain. The client maintains control over eating and exercise.

A client's altered body image is evidenced by claims of "feeling fat" even though the client is emaciated. Which is the appropriate outcome criterion for this client's problem? A. The client will consume adequate calories to sustain normal weight. B. The client will cease strenuous exercise programs. C. The client will perceive an ideal body weight and shape as normal. D. The client will not express a preoccupation with food.

ANS: C The nurse should identify that the appropriate outcome for this client is to perceive an ideal body weight and shape as normal. Additional goals include accepting self based on self-attributes instead of appearance and to realize that perfection is unrealistic.

A client diagnosed with bulimia nervosa has been attending a mental health clinic for several months. Which factor should a nurse identify as an appropriate indicator of a positive client behavioral change? A. The client gains 2 pounds in 1 week. B. The client focuses conversations on nutritious food. C. The client demonstrates healthy coping mechanisms that decrease anxiety. D. The client verbalizes an understanding of the etiology of the disorder.

ANS: C The nurse should identify that when a client uses healthy coping mechanisms that decrease anxiety, positive behavioral change is demonstrated. Stress and anxiety can increase bingeing which is followed by inappropriate compensatory behaviors.

A nurse has been caring for a client diagnosed with post-traumatic stress disorder. What short-term, realistic, correctly written outcome should be included in this client's plan of care? A. The client will have no flashbacks. B. The client will be able to feel a full range of emotions by discharge. C. The client will not require zolpidem (Ambien) to obtain adequate sleep by discharge. D. The client will refrain from discussing the traumatic event.

ANS: C The nurse should include obtaining adequate sleep without zolpidem (Ambien) by discharge as a realistic outcome for this client. Having no flashbacks and experiencing a full range of emotions are long-term not short-term outcomes for this client. Clients are encouraged to discuss the traumatic event.

A client diagnosed with schizoaffective disorder is admitted for social skills training. Which information should be taught by the nurse? A. The side effects of medications B. Deep breathing techniques to decrease stress C. How to make eye contact when communicating D. How to be a leader

ANS: C The nurse should plan to teach the client how to make eye contact when communicating. Social skills, such as making eye contact, can assist clients in communicating needs and maintaining connectedness.

A client diagnosed with brief psychotic disorder tells a nurse about voices telling him to kill the president. Which nursing diagnosis should the nurse prioritize for this client? A. Disturbed sensory perception B. Altered thought processes C. Risk for violence: directed toward others D. Risk for injury

ANS: C The nurse should prioritize the diagnosis risk for violence: directed toward others. A client who hears voices telling him to kill someone is at risk for responding and reacting to the command hallucination. Other risk factors for violence include aggressive body language, verbal aggression, catatonic excitement, and rage reactions.

A morbidly obese client is prescribed an anorexiant medication. The nurse should prepare to teach the client about which medication? A. Diazepam (Valium) B. Dexfenfluramine (Redux) C. Sibutramine (Meridia) OR C. Lorcaserin ( Belviq) D. Pemoline (Cylert)

ANS: C The nurse should teach the client that sibutramine (Meridia) is an anorexiant medication prescribed for morbidly obese clients. The mechanism of action in the control of appetite appears to occur by inhibiting the neutotransmitters serotonin and norepinephrine. Withdrawal from anorexiants can result in rebound weight gain, lethargy, and depression.

A group of nurses are discussing how food is used in their families and the effects this might have on their ability to work with clients diagnosed with eating disorders. Which of these nurses will probably be most effective with these clients? A. The nurse who understands the importance of three balanced meals a day B. The nurse who permits children to have dessert only after finishing the food on their plate C. The nurse who refuses to engage in power struggles related to food consumption D. The nurse who grew up poor and frequently did not have enough food to eat

ANS: C The nurse who refuses to engage in power struggles related to food consumption will probably be most effective when dealing with clients diagnosed with eating disorders. Because of this attitude the nurse recognizes that the real issues have little to do with food or eating patterns. The nurse will be able to focus on the control issues that precipitated these behaviors.

A college student is unable to take a final examination due to severe test anxiety. Instead of studying, the student relieves stress by attending a movie. Which priority nursing diagnosis should a campus nurse assign for this client? A. Noncompliance R/T test taking B. Ineffective role performance R/T helplessness C. Altered coping R/T anxiety D. Powerlessness R/T fear

ANS: C The priority nursing diagnosis for this client is altered coping R/T anxiety. The nurse should assist in implementing interventions that should improve the client's healthy coping skills and reduce anxiety.

A client is diagnosed with schizophrenia. A physician orders haloperidol (Haldol) 50 mg bid, benztropine (Cogentin) 1 mg prn, and zolpidem (Ambien) 10 mg HS. Which client behavior would warrant the nurse to administer benztropine? A. Tactile hallucinations B. Tardive dyskinesia C. Restlessness and muscle rigidity D. Reports of hearing disturbing voices

ANS: C The symptom of tactile hallucinations and reports of hearing disturbing voices would be addressed by an antipsychotic medication such as haloperidol. Tardive dyskinesia, a potentially irreversible condition, would warrant the discontinuation of an antipsychotic medication such as haloperidol. An anticholinergic medication such as benztropine would be used to treat the extrapyramidal symptoms of restlessness and muscle rigidity.

A nurse is seeing a client in an outpatient clinic for treatment of anorexia nervosa. Which is the most appropriate, correctly written short-term outcome for this client? A. The client will use stress-reducing techniques to avoid purging. B. The client will discuss chaos in personal life and be able to verbalize a link to purging. C. The client will gain 2 pounds prior to the next weekly appointment. D. The client will remain free of signs and symptoms of malnutrition and dehydration.

ANS: C The symptoms of anorexia nervosa do not include purging. Correctly written outcomes must be client centered, specific, realistic, measurable, and also include a time frame.

What client information does a nurse need to assess prior to initiating medication therapy with phenelzine (Nardil)? A. The client's understanding of the need for regular blood work B. The client's mood and affect score, using the facility's mood scale C. The client's cognitive ability to understand information about the medication D. The client's access to a support network willing to participate in treatment

ANS: C There are many dietary and medication restrictions when taking Nardil. A client must have the cognitive ability to understand information about the medication and which foods, beverages, and medications to eliminate when taking Nardil.

A client diagnosed with generalized anxiety states, "I know the best thing for me to do now is to just forget my worries." How should the nurse evaluate this statement? A. The client is developing insight. B. The client's coping skills are improving. C. The client has a distorted perception of problem resolution. D. The client is meeting outcomes and moving toward discharge.

ANS: C This client has a distorted perception of how to deal with the problem of anxiety. Clients should be encouraged to openly deal with anxiety and recognize the triggers that precipitate anxiety responses.

A client diagnosed with bulimia nervosa is to receive fluoxetine (Prozac) by oral solution. The medication is supplied in a 100 mL bottle. The label reads 20 mg/5 mL. The doctor orders 60 mg q day. Which dose of this medication should the nurse dispense? A. 25 mL B. 20 mL C. 15 mL D. 10 mL

ANS: C Twenty mg of Prozac multiplied by three results in the calculated 60 mg daily dose ordered by the physician. Each 5 mL contains 20 mg. Five mL multiplied by three equals the liquid dosage of 15 mL.

The nurse is providing counseling to clients diagnosed with major depressive disorder. The nurse chooses to assess and attempt to modify the negative thought patterns of these clients. The nurse is functioning under which theoretical framework? A. Psychoanalytic theory B. Interpersonal theory C. Cognitive theory D. Behavioral theory

ANS: C When a nurse assesses and attempts to modify negative thought patterns related to depressive symptoms, the nurse is using a cognitive theory framework.

A client diagnosed with schizophrenia takes an antipsychotic agent daily. Which assessment finding should a nurse immediately report to the clients attending psychiatrist? A. Respirations of 22 beats/minute B. Weight gain of 8 pounds in 2 months C. Temperature of 104F (40C) D. Excessive salivation

ANS: C When assessing a client diagnosed with schizophrenia who takes an antipsychotic agent daily, the nurse should immediately address a temperature of 104F (40C). A temperature this high can be a symptom of the rare but life-threatening neuroleptic malignant syndrome.

A nurse responsible for conducting group therapy on an eating disorder unit schedules the sessions immediately after meals. Which is the rationale for scheduling group therapy at this time? A. To shift the clients' focus from food to psychotherapy B. To prevent the use of maladaptive defense mechanisms C. To promote the processing of anxiety associated with eating D. To focus on weight control mechanisms and food preparation

ANS: C When the nurse schedules group therapy immediately after meals, the nurse is addressing the emotional issues related to eating disorders that must be resolved if these maladaptive responses are to be eliminated.

Sertraline (Zoloft) has been prescribed for a client complaining of poor appetite, fatigue, and anhedonia. Which consideration should the nurse recognize as influencing this prescriptive choice? A. Zoloft is less expensive for the client. B. Zoloft is extremely sedating and will help with sleep disturbances. C. Zoloft has less adverse side effects than other antidepressants. D. Zoloft begins to improve depressive symptoms quickly.

ANS: C Zoloft is a selective serotonin reuptake inhibitor (SSRI) that has a relatively benign side effect profile as compared with other antidepressants.

Why are behavior modification programs the treatment of choice for clients diagnosed with eating disorders? A. These programs help clients correct distorted body image. B. These programs address underlying client anger. C. These programs help clients manage uncontrollable behaviors. D. These programs allow clients to maintain control.

ANS: D Behavior modification programs are the treatment of choice for clients diagnosed with eating disorders because these programs allow clients to maintain control. Issues of control are central to the etiology of these disorders. Behavior modification techniques aid in restoring healthy body weight.

A nurse is planning care for a child who is experiencing depression. Which medication is approved by the U.S. Food and Drug Administration (FDA) for the treatment of depression in children and adolescents? A. Paroxetine (Paxil) B. Sertraline (Zoloft) C. Citalopram (Celexa) D. Fluoxetine (Prozac)

ANS: D Fluoxetine (Prozac) is FDA approved for the treatment of depression in children and adolescents. Fluoxetine is a selective serotonin reuptake inhibitor (SSRI) used in the treatment of depression. All antidepressants carry an FDA warning for increased risk of suicide in children and adolescents.

Which client statement expresses a typical underlying feeling of clients diagnosed with major depressive disorder? A. "It's just a matter of time and I will be well." B. "If I ignore these feelings, they will go away." C. "I can fight these feelings and overcome this disorder." D. "Nothing will help me feel better."

ANS: D Hopelessness and helplessness are typical symptoms of clients diagnosed with major depressive disorder. Depressive symptoms are often described as anger turned inward.

A nurse is caring for four clients taking various medications including imipramine (Tofranil), doxepine (Sinequan), ziprasidone (Geodon), and tranylcypromine (Parnate). The nurse orders a special diet for the client receiving which medication? A. Tofranil B. Senequan C. Geodon D. Parnate

ANS: D Hypertensive crisis occurs in clients receiving monoamine oxidase inhibitor (MAOI) who consume foods or drugs high in tyramine content.

A nurse is assessing a client diagnosed with paranoid schizophrenia. The nurse asks the client, "Do you receive special messages from certain sources, such as the television or radio?" Which potential symptom of this disorder is the nurse assessing? A. Thought insertion B. Paranoid delusions C. Magical thinking D. Delusions of reference

ANS: D The nurse is assessing for the potential symptom of delusions of reference. A client who believes that he or she receives messages through the radio is experiencing delusions of reference. When a client experiences these delusions, he or she interprets all events within the environment as personal references.

A client diagnosed with seasonal affective disorder (SAD) states, "I've been feeling 'down' for 3 months. Will I ever feel like myself again?" Which reply by the nurse will best assess this client's symptoms. A. "Have you been diagnosed with any physical disorder within the last 3 months?" B. "Have you experienced any traumatic events that triggered this mood change?" C. "People who have seasonal mood changes often feel better when spring comes." D. "Help me understand what you mean when you say, 'feeling down'?"

ANS: D The nurse is using a clarifying statement in order to gather more details related to this client's mood. The diagnosis of SAD is not associated with a traumatic event.

A client diagnosed with major depressive disorder states, Ive been feeling down for 3 months. Will I ever feel like myself again? Which reply by the nurse will best assess this clients affective symptoms? A. Have you been diagnosed with any physical disorder within the last 3 months? B. Have you ever felt this way before? C. People who have mood changes often feel better when spring comes. D. Help me understand what you mean when you say, feeling down?

ANS: D The nurse is using a clarifying statement in order to gather more details related to this clients mood.

A nurse assesses a client suspected of having major depressive disorder. Which client symptom would eliminate this diagnosis? A. The client is disheveled and malodorous. B. The client refuses to interact with others. C. The client is unable to feel any pleasure. D. The client has maxed-out charge cards and exhibits promiscuous behaviors.

ANS: D The nurse should assess that a client who has maxed-out credit cards and exhibits promiscuous behavior would be exhibiting manic symptoms. According to the DSM-IV-TR, these symptoms would rule out the diagnosis of major depressive disorder.

A client is diagnosed with persistent depressive (dysthymia) disorder. Which should a nurse classify as an affective symptom of this disorder? A. Social isolation with a focus on self B. Low energy level C. Difficulty concentrating D. Gloomy and pessimistic outlook on life

ANS: D The nurse should classify a gloomy and pessimistic outlook on life as an affective symptom of dysthymia. Symptoms of depression can be described as alterations in four areas of human functions: affective, behavioral, cognitive, and physiological. Affective symptoms are those that relate to the mood.

A client is diagnosed with dysthymic disorder. Which should a nurse classify as an affective symptom of this disorder? A. Social isolation with a focus on self B. Low energy level C. Difficulty concentrating D. Gloomy and pessimistic outlook on life

ANS: D The nurse should classify a gloomy and pessimistic outlook on life as an affective symptom of dysthymic disorder. Symptoms of depression can be described as alterations in four areas of human functions: affective, behavioral, cognitive, and physiological.

A client is newly diagnosed with obsessive-compulsive disorder and spends 45 minutes folding clothes and rearranging them in drawers. Which nursing intervention would best address this client's problem? A. Distract the client with other activities whenever ritual behaviors begin. B. Report the behavior to the psychiatrist to obtain an order for medication dosage increase. C. Lock the room to discourage ritualistic behavior. D. Discuss the anxiety-provoking triggers that precipitate the ritualistic behaviors.

ANS: D The nurse should discuss with the client the anxiety-provoking triggers that precipitate the ritualistic behavior. If the client is going to be able to avoid the anxiety, he or she must first learn to recognize precipitating factors. Attempting to distract the client, seeking medication increase, and locking the client's room are not appropriate interventions because they do not help the client recognize anxiety triggers.

A client diagnosed with chronic schizophrenia presents in an emergency department (ED) with uncontrollable tongue movements, stiff neck, and difficulty swallowing. The nurse would expect the physician to recognize which condition and implement which treatment? A. Neuroleptic malignant syndrome and treat by discontinuing antipsychotic medications B. Agranulocytosis and treat by administration of clozapine (Clozaril) C. Extrapyramidal symptoms and treat by administration of benztropine (Cogentin) D. Tardive dyskinesia and treat by discontinuing antipsychotic medications

ANS: D The nurse should expect that an ED physician would diagnose the client with tardive dyskinesia and discontinue antipsychotic medications. Tardive dyskinesia is a condition of abnormal involuntary movements of the mouth, tongue, trunk, and extremities that can be an irreversible side effect of typical antipsychotic medications.

A client diagnosed with an obsessive-compulsive disorder spends hours bathing and grooming. During a one-on-one interaction, the client discusses the rituals in detail but avoids any feelings that the rituals generate. Which defense mechanism should the nurse identify? A. Sublimation B. Dissociation C. Rationalization D. Intellectualization

ANS: D The nurse should identify that the client is using the defense mechanism of intellectualization when discussing the rituals of obsessive-compulsive disorder in detail while avoiding discussion of feelings. Intellectualization is an attempt to avoid expressing emotions associated with a stressful situation by using the intellectual processes of logic, reasoning, and analysis.

During an admission assessment, a nurse notes that a client diagnosed with schizophrenia has allergies to penicillin, prochlorperazine (Compazine), and bee stings. Based on this assessment data, which antipsychotic medication would be contraindicated? A. Haloperidol (Haldol), because it is used only in elderly patients B. Clozapine (Clozaril), because of a cross-sensitivity to penicillin C. Risperidone (Risperdal), because it exacerbates symptoms of depression D. Thioridazine (Mellaril), because of cross-sensitivity among phenothiazines

ANS: D The nurse should know that thioridazine (Mellaril) would be contraindicated because of cross-sensitivity among phenothiazines. Prochlorperazine (Compazine) and thioridazine are both classified as phenothiazines.

How would a nurse differentiate a client diagnosed with panic disorder from a client diagnosed with generalized anxiety disorder (GAD)? A. GAD is acute in nature, and panic disorder is chronic. B. Chest pain is a common GAD symptom, whereas this symptom is absent in panic disorders. C. Hyperventilation is a common symptom in GAD and rare in panic disorder. D. Depersonalization is commonly seen in panic disorder and absent in GAD.

ANS: D The nurse should recognize that a client diagnosed with panic disorder experiences depersonalization, whereas a client diagnosed with GAD would not. Depersonalization refers to being detached from oneself when experiencing extreme anxiety.

A cab driver, stuck in traffic, suddenly is lightheaded, tremulous, diaphoretic, and experiences tachycardia and dyspnea. An extensive workup in an emergency department reveals no pathology. Which medical diagnosis is suspected, and what nursing diagnosis takes priority? A. Generalized anxiety disorder and a nursing diagnosis of fear B. Altered sensory perception and a nursing diagnosis of panic disorder C. Pain disorder and a nursing diagnosis of altered role performance D. Panic disorder and a nursing diagnosis of anxiety

ANS: D The nurse should suspect that the client has exhibited signs/symptoms of a panic disorder. The priority nursing diagnosis should be anxiety. Panic disorder is characterized by recurrent, sudden onset panic attacks in which the person feels intense fear, apprehension, or terror.

A confused client has recently been prescribed sertraline (Zoloft). The client's spouse is taking paroxetine (Paxil). The client presents with restlessness, tachycardia, diaphoresis, and tremors. What complication does a nurse suspect and what could be its possible cause? A. Neuroleptic malignant syndrome caused by ingestion of two different seratonin reuptake inhibitors (SSRIs) B. Neuroleptic malignant syndrome caused by ingestion of an SSRI and a monoamine oxidase inhibitor (MAOI) C. Serotonin syndrome caused by ingestion of an SSRI and an MAOI D. Serotonin syndrome caused by ingestion of two different SSRIs

ANS: D The nurse should suspect that the client is suffering from serotonin syndrome possibly caused by ingesting two different SSRIs (Zoloft and Paxil). Symptoms of serotonin syndrome include confusion, agitation, tachycardia, hypertension, nausea, abdominal pain, myoclonus, muscle rigidity, fever, sweating, and tremor.

A potential Olympic figure skater collapses during practice and is hospitalized for severe malnutrition. Anorexia nervosa is diagnosed. Which client statement best reflects the underlying etiology of this disorder? A. "Skaters need to be thin to improve their daily performance." B. "All the skaters on the team are following an approved 1,200-calorie diet." C. "When I lose skating competitions, I also lose my appetite." D. "I am angry at my mother. I can only get her approval when I win competitions."

ANS: D This client statement reflects the underlying etiology of anorexia nervosa. The client is expressing feelings about family dynamics that may have influenced the development of this disorder. Families who are overprotective and perfectionistic can contribute to a family member's development of anorexia nervosa.

A client diagnosed with anorexia nervosa stopped eating 5 months ago and lost 25% of total body weight. Which subjective client response would the nurse assess to support this medical diagnosis? A. "I do not use any laxatives or diuretics to lose weight." B. "I am losing lots of hair. It's coming out in handfuls." C. "I know that I am thin, but I refuse to be fat!" D. "I don't know why people are worried. I need to lose this weight."

ANS: D When the client states, "I don't know why people are worried. I need to lose this weight," the client is exhibiting the subjective response of ineffective denial. This client is minimizing symptoms and is unable to admit impact of the disease on life patterns. The client does not perceive personal relevance of symptoms or danger.

Psychological domain nursing disgnosis for schizophrenia

Acute confusion Disturbed thought process Chronic low self esteem Ineffective coping Knowledge deficit

A client diagnosed with Bipolar I Disorder is distraught over insomnia experienced over the last 3 nights and a 12-pound weight loss over the past 2 weeks. Which should be this client's priority nursing diagnosis? A. Knowledge deficit R/T bipolar disorder AEB concern about symptoms B. Altered nutrition: less than body requirements R/T hyperactivity AEB weight loss C. Risk for suicide R/T powerlessness AEB insomnia and anorexia D. Altered sleep patterns R/T mania AEB insomnia for the past 3 nights

Altered nutrition: less than body requirements R/T hyperactivity AEB weight loss

A client diagnosed with bipolar I disorder is distraught over insomnia experienced over the last 3 nights and a 12-pound weight loss over the past 2 weeks. Which should be this client's priority nursing diagnosis?

Altered nutrition: less than body requirements R/T hyperactivity AEB weight loss

Some patients with schizophrenia express lack of insight or awareness that there is anything wrong or that any disorder is present. This symptom is referred to as _____________.

Anosognosia This symptom is often apparent when a client is asked what prompted admission to the hospital. A response such as "for some reason, the police just came over and told me I had to go to the hospital," is evidence of anosognosia. It is considered a symptom of the illness rather than a defense mechanism. A comparable symptom occurs following brain damage.

Which carries a warning label stating that the use of the medication increases risk for suicidal thoughts and behaviors? A. Antipsychotics B. Antiepileptics C. Mood stabilizers D. Anxiolytics

Anxiolytics

Which of the following instructions regarding lithium therapy should be included in a nurse's discharge teaching? Select all that apply.

Avoid excessive use of beverages containing caffeine. Maintain a consistent sodium intake. Consume at least 2,500 to 3,000 mL of fluid per day.

Several types of delusions may occur in an individual with schizophrenia. Which of the following types of delusion places the patient at greatest risk for agitation or aggression? A. delusions of grandeur B. delusions of persecution C. delusions of reference D. nihilistic delusions

B. delusions of persecution In delusions of persecution an individual falsely believes he or she is being threatened or persecuted in some way. This carries a high risk for increasing the individual's agitation and possibly aggression in protective efforts.

In planning care to reinforce reality for a client diagnosed with schizophrenia, the nurse should include which interventions? A. explore the client's expressions of distorted thinking B. discuss perceptions and thinking that are in touch with reality C. Encourage the client to share delusional thinking in group discussions. D. ask the client why distorted thinking and bizarre behavior have occurred.

B. discuss perceptions and thinking that are in touch with reality Discussing reality=based perceptions and thinking will assist the client to maintain orientation and will promote organized thinking.

The family of a patient with schizophrenia requests information about Assertive Community Treatment (ACT). Which of the following responses by the nurse are consistent with this treatment model? (Select all that apply) A. "this model of treatment is based in the hospital and provides group education about how to assert oneself in the community." B. "this is a program of case management that takes a team approach in providing comprehensive community-based psychiatric services." C. This model is designed to meet the needs of people with conditions ranging from mild depression to severe and persistent illnesses such as schizophrenia." D. "One of the primary goals of ACT is to lessen the family's burden of providing care."

B. "this is a program of case management that takes a team approach in providing comprehensive community-based psychiatric services." D. "One of the primary goals of ACT is to lessen the family's burden of providing care." Feedback 1: This response demonstrates a lack of understanding of ACT. ACT is a community-based treatment model that focuses on comprehensive management of needs for patients with severe and persistent mental illnesses like schizophrenia. Feedback 4: NAMI (2012) identifies primary goals for ACT, one of which is to lessen the family's burden for providing care. ACT recognizes that patients with severe, persistent mental illness require many services beyond what one resource or the family can provide exclusively

The nurse is interviewing a client on the psychiatric unit. The client tilts his head to the side, stops talking in midsentence, and listens intently. the nurse recognizes these behaviors as a symptom of the client's illness. The most appropriate nursing intervention for this symptom is to: A. Ask the client to describe his physical symptoms B. Ask the client to describe what he is hearing C. Administer a dose of benzotropine D. Call the physician for additional orders

B. Ask the client to describe what he is hearing

The primary goal in working with an actively psychotic, suspicious client would be to: A. Promote interaction with others B. Decrease his anxiety and increase trust C. Improve this relationship with his parents D. Encourage participation in therapy activities

B. Decrease his anxiety and increase trust

A client diagnosed with schizophrenia manifests the symptom of mutism. Which nursing intervention would assist the client in communicating with others? A. Providing assistance with self-care needs B. Using clear, concrete statements C. Conveying acceptance of the client's need for false beliefs D. attempting to decode incomprehensible communication patterns.

B. Using clear, concrete statements The use of clear, concrete statement shows the client what is expected. Because clients diagnosed with schizophrenia experience concrete thinking, explanations must be provided at the client's concrete level of comprehension

A client diagnosed with schizophrenia hears another patient say, "You'll be tied up for another hour." and becomes agitated because he interprets that to mean he will literally be tied up. Which cognitive symptom of schizophrenia is this client manifesting? A. nihilistic delusions B. concrete thinking C. circumstantiality D. perseveration

B. concrete thinking Concrete thinking is manifested by literal interpretation of abstract or figurative ideas. This symptom may be present in schizophrenia and is believed to represent regression to an earlier level of cognitive development

A nursing home resident who has been taking antipsychotic medications for several months complains to the nurse of a stiff neck and difficulty swallowing. These symptoms are indicative of which condition? A. dysphonia B. tardive dyskinesia C. akathisia D. echolalia

B. tardive dyskinesia Tardive dyskinesia is a syndrome characterized by abnormal, involuntary movements, including bizarre facial and tongue movements, a stiff neck, and/or difficulty swallowing. This condition may occur as an adverse effect of long-term therapy with antipsychotic medications

Which of the following explanations should a nurse include when teaching parents why is it difficult to diagnose a child or adolescent exhibiting symptoms of bipolar disorder? Select all that apply.

Bipolar symptoms are similar to attention deficit-hyperactivity disorder symptoms. Children are naturally active, energetic, and spontaneous.

Antipsychotics

Blockage of dopamine transmission Used to decrease agitation and psychotic symptoms of schizophrenia and other psychotic disorders Second generation(atypical) effective in treating negative and positive symptoms Monitoring and admission: 1-2 weeks to effect a change in symptoms, trial of 6-12 weeks before med change Clozapine used when no other second generation agent effective Side effects: orthostatic hypotenstion, weight gain, reduced seizure threshold, decreased WBC, extrapyramidal symptoms, tar dive dyskinesia, neuroleptic malignant syndrome - extrapyramidal symptoms(EPS): psudeoparkinsonism, akinesia, akathisia, dystonia, oculogyric crisis = antiparkinsonian agents to be prescribed, anticholinergics, benzotropine(anticholinergic that blocks cholinergic activity in CNS responsible for EPS) - neuroleptic malignant syndrome: severe muscle rigidity, elevated temperature with rapidly accelerating cascade of symptoms = withhold med, dopamine agonist(bromocriptine), muscle relaxants(dantrolene or benzodiazepine) - anticholingeric crisis: hot, blind, mad, dry = discontinue med, physostigmine, gastric lavage/charcoal/catharsis for intentional overdoses

A client is admitted with a diagnosis of schizoaffective disorder. Which symptoms are characteristic of this diagnosis? A. strong ego boundaries and abstract thinking B. acute dystonias and tardive dyskinesia C. Altered mood and thought disturbances D. substance abuse and cachexia

C. Altered mood and thought disturbances The characteristic symptoms of schizoaffective disorder are a combination of alterations in mood (mania or depression) and thought

A client who has been taking chlorpromazine (Thorazine) for several months presents in the ED with extrapyramidal symptoms (EPS) of restlessness, drooling and tremors. What medication will the nurse expect the physician to order? A. Paroxetine (Paxil) B. Carbamazepine (Tegretol) C. Benztropine (Cogentin) D. Lorazepam (Ativan)

C. Benztropine (Cogentin) Benztropine is an anticholinergic medication that blocks cholinergic activity in the CNS, which is responsible for EPS. Anticholinergics are the drugs of choice to treat extrapyramidal symptoms associated with antipsychotic mediations

A client who has been taking chlorpromazine (Thorazine) for several months presents in the ED with extrapyramidal symptoms of restlessness, drooling, and tremors. What medication will the nurse expect the physician to order? A. Paroxetine (Paxil) B. Carbamazepine (Tegretol) C. Benztropine (Cogentin) D. Lorazepam (Ativan)

C. Benztropine (Cogentin) Cogentin is an anticholinergic medication that blocks cholinergic activity in the central nervous system, which is responsible for extrapyramidal symptoms. This is the drug of choice to treat extrapyramidal symptoms associated with antipsychotic medications.

A client is admitted with a diagnosis of brief psychotic disorder with catatonic features. Which symptoms are associated with the catatonic specifier? A. Strong ego boundaries and abstract thinking B. Ataxia and akinesia C. Stupor, muscle rigidity, and negativism D. substance abuse and cachexia

C. Stupor, muscle rigidity, and negativism Symptoms associated with the catatonic specifier include stupor and muscle rigidity or excessive, purposeless motor activity. Waxy flexibility, negativism, echolalia, and echopraxia are also common behaviors

The nurse is caring for a client with schizophrenia. Orders from the physician include 100 mg chlorpromazine IM STAT and then 50 mg PO bid; 2 mg benztropine PO bid PRN. Why is chlorpromazine ordered? A. To reduce extrapyramidal symptoms B. To prevent neuroleptic malignant syndrome C. To decrease psychotic symptoms D. To induce sleep

C. To decrease psychotic symptoms

The nurse is interviewing a client on the psychiatric unit. The client tilts is head to the side, stops talking in midsentence, and listens intently. The nurse recognizes from these signs that the client is likely experiencing: A. somatic delusions B. catatonic stupor C. auditory hallucinations D. pseudoparkinsonism

C. auditory hallucinations

A client is experiencing paranoia and states, "the FBI and phone company are plotting against me." Which charting entry best describes this client's symptom? A. experiencing delusions of grandeur B. experiencing erotomanic delusions C. experiencing delusions of persecution D. experiencing somatic delusions

C. experiencing delusions of persecution Individuals experiencing delusions of persecution feel that they are being threatened and believe that others have harmful intentions. The client in the question believes that the FBI and the phone company are plotting harm.

The client hears the word "match". the client replies, "A match. I like matches. They are the light of the world. God will light the world. Let your light so shine." Which communication pattern does the nurse identify? A. word salad B. clang association C. Loose association D. ideas of reference

C. loose association Loose association is characterized by communication in which ideas shift from one unrelated topic to another. The situation in the question represents this communication pattern.

The client hears the word "match". The client says, "A match. Tomorrow is the end of the world. Nothing is better than hot coffee." Which communication pattern does the nurse identify? A. word salad B. clang association C. loose association D. ideas of reference

C. loose association Loose association is characterized by communication in which ideas shit from one unrelated topic to another. The situation in the question clearly represents this communication pattern

The nurse is providing care for an emaciated client experiencing an acute phase of catatonic stupor. Which nursing intervention would take priority when meeting this client's needs? A. minimize attempts to communicate with the client B. assist the client to ambulate C. provide nutrient-dense foods and beverages D. place the patient is seclusion for safety

C. provide nutrient-dense foods and beverages Nutrition is an essential consideration for a client with catatonic stupor. The emaciated client in the question is suffering from malnutrition. The nurse must prioritize this basic physical need.

A client has been admitted to the inpatient psychiatric unit and is manifesting mutism. His diagnosis is schizophrenia with catotonia. What would the nurse expect to observe? A. frenzied and purposeless movements B. exaggerated suspiciousness C. stuporous withdrawal D. sexual preoccupation

C. stuporous withdrawal The client's mutism indicates catatonic stupor. This client would be noted to have extreme psychomotor retardation, and efforts to move the individual may be met with bodily resistence

A client diagnosed with schizophrenia is experiencing disorganized thinking. Which technique should the nurse use to promote communication? A. giving broad openings B. probing C. verbalizing the implied D. using open-ended questions

C. verbalizing the implied When working with clients who have greatly impaired communication ability, the nurse can use the technique of verbalizing the implied. By putting into words what the client may be experiencing, the nurse helps the client to organize his or her thinking.

52. A client has been diagnosed with major depression. The psychiatrist prescribes imipramine (Tofranil). Which of the following medication information should the nurse include in discharge teaching? Select all that apply. 1) "The medication may cause dry mouth." 2) "The medication may cause urinary incontinence." 3) "The medication should not be discontinued abruptly." 4) "The medication may cause photosensitivity." 5) "The medication may cause nausea."

Correct 1: Dry mouth can occur with all antidepressants, including imipramine. Feedback 2: Urinary retention, not incontinence, may occur when taking imipramine. Correct 3: Antidepressants such as imipramine must be tapered and not stopped abruptly. Correct 4: Tricyclic antidepressants such as imipramine can cause photosensitivity, whereas other types of antidepressants do not. Therefore, the client must be educated specifically about this potential side effect. Correct 5: Nausea can occur with all antidepressants, including imipramine.

63. The nurse is conducting an assessment for Leroy, a 65-year-old man who presented at the health clinic with complaints of depression. He lists several medications he has been taking. Of the following medications on his list, which are known to produce a depressive syndrome? Select all that apply. 1) Prednisone 2) Cimetidine (Tagamet) 3) Ampicillin 4) Ibuprofen (Advil) 5) Aspirin

Correct 1: Prednisone is a steroid medication that can produce depression. Correct 2: Cimetidine is an anti-ulcer medication that can produce depression. Correct 3: Ampicillin is an antibacterial medication that can produce depression. Correct 4: Ibuprofen is an analgesic/anti-inflammatory medication that can produce depression. Feedback 5: Aspirin has not been associated with producing depression.

51. A client diagnosed with major depression is being discharged from the hospital with a prescription for fluoxetine (Prozac). The nurse's discharge teaching should include which of the following? Select all that apply. 1) "It may take a few weeks before you begin to feel better; however, continue taking Prozac as prescribed." 2) "Make sure that you follow up with scheduled outpatient psychotherapy." 3) "If significant mood elevation is noted, your psychiatrist may discontinue this medication within 6 months to a year." 4) "You should avoid foods with tyramine, including beer, beans, processed meats, and red wine." 5) "You can discontinue the Prozac when you are feeling better."

Correct 1: The nurse should inform the client that it is important to take Prozac as prescribed and that the therapeutic effect can take up to 4 weeks to be realized. Correct 2: Along with medication compliance, the nurse should also stress the importance of follow-up psychotherapy. Correct 3: The nurse should advise the client to discontinue the medication only under a doctor's supervision. Although the medication may be tapered and stopped after 6 months, there is a risk for further depressive episodes. Feedback 4: Avoidance of foods with tyramine would hold true if the client were taking an MAOI, not a selective serotonin reuptake inhibitor, such as Prozac. Feedback 5: The client should be advised to not stop taking Prozac abruptly. To do so might produce withdrawal symptoms such as nausea, vertigo, insomnia, headache, malaise, and nightmares.

To deal with a client's hallucinations therapeutically, which nursing intervention should be implemented? a) reinforce the perceptual distortions until the client develops new defenses b) provide an unstructured environment c) avoid making connections between anxiety-producing situations and hallucinations d) distract the client's attention

Correct answer: D The nurse should first empathize with the client by focusing on feelings generated by the hallucination, present objective reality, and then distract or redirect the client to reality-based activities.

Clint, a client on the psychiatric unit, has been diagnosed with schizophrenia. He begins to tell the nurse about how the CIA is looking for him and will kill him if they find him. The most appropriate response by the nurse is: A. "That's ridiculous, Clint. No one is going to hurt you." B. "The CIA isn't interested in people like you, Clint." C. "Why do you think the CIA wants to kill you?" D. "I know you believe that, Clint, but it's really hard for me to believe."

D. "I know you believe that, Clint, but it's really hard for me to believe."

A client is being discharged on haloperidol (Haldol). Which teaching should the nurse include about the medication? A. "If you forget to take your morning dose of Haldol, double the dose at bedtime." B. "Limit your alcohol intake to no more than 3 ounces per day. " C. "When you go home, sit outside and enjoy the sunshine." D. "Do not stop taking Haldol abruptly."

D. "Do not stop taking Haldol abruptly." The client should be taught not to stop taking Haldol abruptly after long-term use. To do so might produce withdrawal symptoms, such as N/V, dizziness, gastritis, headache, tachycardia, insomnia and/or tremulousness

Which tool should the nurse use to differentiate occasional spontaneous behaviors of children from behaviors associated with bipolar disorder? A. Risky Activity tool B. FIND tool C. Consensus Committee tool D. Monotherapy tool

FIND tool

Which medication does the nurse determine will give the client the most immediate relief from neuroleptic-induced extrapyramidal side effects? A. lorazepam (Ativan), 1 mg PO B. Diazepam (Valium), 5 mg PO C. Haloperidol (Haldol), 2 mg IM D. Benztropine (Cogentin), 2 mg PO

D. Benztropine (Cogentin), 2 mg PO The symptoms of neuroleptic-induced extrapyramidal side effects include tremors, chorea, dystonia, akinesia, and akahesia. Congentin, 1-4 mg given once or twice daily, is the drug of choice to treat these symptoms.

The nurse is caring for a client with schizophrenia. Orders from the physician include 100 mg chlorpromazine IM STAT, and then 50 mg PO bid; 2 mg benztropine PO bid PRN. Because benztropine was ordered on a PRN basis, which of the following assessments by the nurse would convey a need for this medication? A. The client's level of agitation increases B. The client complains of a sore throat C. The client's skin has a yellowish cast. D. The client develops tremors and a shuffling gait.

D. The client develops tremors and a shuffling gait.

The primary focus of family therapy for clients with schizophrenia and their families is: A. to discuss concrete problem solving and adaptive behaviors for coping with stress B. To introduce the family to others with the same problem C. To keep the client and family in touch with the health care system D. To promote family interaction and increase understanding of the illness

D. To promote family interaction and increase understanding of the illness

To deal with a client's hallucinations therapeutically, which nursing intervention should be implemented? A. Reinforce the perceptual distortions until the client develops new defenses B. Provide an unstructured environment C. Avoid making connections between anxiety-producing situations and hallucinations D. distract the client's attention

D. distract the client's attention The nurse should first empathize with the client by focusing on feelings generated by the hallucination, present objective reality, and then distract or redirect the client to reality-based activities.

To deal with a client's hallucinations therapeutically, which nursing intervention should be implemented? A. reinforce the perceptual distortions until the client develops new defenses. B. Provide an unstructured environment C. avoid making connections between anxiety-producing situations and hallucinations D. use empathic listening and redirect the client's attention to reality-based interaction

D. use empathic listening and redirect the client's attention to reality-based interaction The nurse should first empathize with the client by focusing on feelings generarted by the hallucination, present objective reality, and then redirect the client to reality-based activites

A client diagnosed with bipolar disorder, who has taken lithium carbonate (Lithane) for 1 year, presents in an emergency department with severe diarrhea, blurred vision, and tinnitus. How should the nurse interpret these symptoms?

Symptoms indicate lithium carbonate toxicity.

Gender differences in schizophrenia

Earlier diagnosis and poorer prognosis in men

Negative symptoms of schizophrenia

Emotions and behaviors that should be present but are diminished Avolition, ambivalence, flat affect - inappropriate affect: emotions are incongruent with the circumstances - bland affect: weak emotional tone - flat affect: appears to be void of emotional tone - apathy: disinterest in the environment - avolition: inability to initiate goal-directed activity

Positive symptoms of schizophrenia

Excessive or distorted thoughts and perceptions Delusions(thoughts) and hallucinations(sensory) - loose associations: shift of ideas from one unrelated topic to another - neologisms: made up words that have meaning only to the person who invents them - clang associations: choice of words is governed by sound - word salad: group of words put together in a random fashion - circumstantiality: delay in reaching the point of a communication because of unnecessary and tedious details - tangentiality: inability to get to the point of communication due to introduction of many new topics - perservation: persistent repetition of the same word or idea in response to different questions - echolalia: repeating words or phrases spoken by another Thinking: echolalia, circumstantiality, loose associations, tangentiality, flight of ideas, word salad, neologisms, paranoias, referential thinking, autistic thinking, concrete thinking, clang association, stilted language, pressured speech Behaviors: aggression, agitation, Catatonia, catatonic excitement, Echopraxia, regressed behavior, hyper vigilance, waxy flexibility

Delusional disorder

Existence of prominent, nonbizarre delusions -erotomanic type, grandiose type, jealous type, persecutory type, somatic type, mixed type

53. Emily has been receiving treatment for major depressive disorder over several weeks. She is taking an antidepressant and attending cognitive behavioral therapy group once a week. When the nurse evaluates her progress in treatment, which of the following are indications that the depression is improving? Select all that apply. 1) Emily is taking the antidepressant medication as ordered. 2) Emily is expressing hope that she can return to her university classes soon and continue her education. 3) Emily demonstrates ability to make decisions concerning her own self-care. 4) Emily reports that suicide ideas have subsided. 5) Emily is engaging in activities that she enjoys.

Feedback 1: Adherence to the medication regime does not presume effectiveness. More relevant indications would be the patient's report of improved mood, improved sleep and rest, and increase in energy. Correct 2: Hopelessness is a characteristic symptom in major depressive disorder, and a return to expressing hopefulness is an indicator of improvement. Correct 3: Indecisiveness is a symptom in depression, and a return to the ability to make decisions is an indication of improvement. Correct 4: Suicide ideas can be pervasive and troubling symptoms in depression. When they begin to abate, it may be an indication that the depression is lifting. Correct 5: One of the symptoms in major depressive disorder is lack of interest in activities that one used to enjoy. The return of interest in activities and in social interaction are indications that the depression is abating.

58. Janice is diagnosed with major depressive disorder and is beginning to participate in a cognitive therapy group. As the nurse is orienting Janice to the group, which of the following statements about cognitive therapy are accurate? Select all that apply. 1) Cognitive therapy is designed to focus on emotional dysregulation. 2) Cognitive distortions, such as negative expectations about oneself, serve as the basis for depression. 3) Cognitive therapy focuses on altering mood by changing the way one thinks. 4) Cognitive distortions arise out of a defect in cognitive development. 5) Cognitive therapy explores pent-up rage that has been turned against oneself because of identification with the loss of a loved object.

Feedback 1: In cognitive therapy the focus is on cognitive distortions. Emotional dysregulation is the central focus of dialectical behavior therapy. Correct 2: Beck et al. (1979) postulated that negative and irrational thinking contribute to depression. These are referred to as cognitive distortions. Correct 3: A primary assumption in cognitive therapy is that changing the way one thinks will change one's mood. Specifically, developing patterns of more rational and positive thinking will improve one's mood. Correct 4: In cognitive theory, it is assumed that cognitive distortions arise from a defect in cognitive development, which culminates in an individual thinking that he or she is worthless, inadequate, and rejected by others. These patterns of thinking need to be corrected to promote a positive change in mood. Feedback 5: The concept of rage turned inward is based in psychoanalytical theory, not cognitive theory.

48. Susan is being seen in the emergency department. Her sister brought her in with concern that Susan is depressed and might be suicidal. Which of the following questions are priorities for the nurse to ask when assessing for suicide risk? Select all that apply. 1) "Why are you feeling depressed and suicidal?" 2) "Are you having thoughts of hurting or killing yourself?" 3) "When you have these thoughts, do you have a plan in mind?" 4) "Do you ever feel like you want to hurt someone else?" 5) "Are you currently using any drugs or alcohol?"

Feedback 1: This question is not relevant and, in general, is nontherapeutic. It challenges the client and does not identify level of suicide risk. Correct 2: Asking this question elicits information about whether the client is having suicide ideation and promotes further assessment of how often, how intrusive, and how intentional the person perceives these ideas to be. Correct 3: Asking this question allows the nurse to assess whether the client's thoughts have become more specific and intentional. It also allows the nurse to assess the lethality of means, which is important information in assessing suicide risk. Additional assessment should include an assessment of whether or not the client has access to the identified means for attempting suicide. Feedback 4: This question is directed toward assessing other directed violence and/or homicidal ideation rather than suicidal ideation. Correct 5: This is a priority question, since evidence supports that substance abuse by people with depression and suicidal ideation increases the risk for suicide.

Familial differences in schizophrenia

First-degree biologic relatives with greater risk

A newly admitted client is diagnosed with Bipolar Disorder: Manic Episode. Which symptom related to altered thought is the nurse most likely to assess? A. Pacing B. Flight of ideas C. Lability of mood D. Irritability

Flight of ideas

A newly admitted client is diagnosed with bipolar disorder: manic episode. Which symptom related to altered thought is the nurse most likely to assess?

Flight of ideas

Predisposing factors of schizophrenia

Genetics Biochemical: excess dopamine Physiological: viral infection, anatomical abnormalities - anatomical abnormalities: larger lateral and third ventricles, smaller total brain volume Environmental: poverty, stressful life events Theoretical integration and transaction model: biological based disease of which the onset is influenced by internal and external environment

Factors associated with positive prognosis of schizophrenia

Good premorbid functioning Later age onset Female gender Abrupt onset precipitated by stressful event Associated mood disturbance Brief duration of active phase symptoms Minimal residual symptoms A sense of structural brain abnormalities Normal neurological functioning No family history of mood disorder No family history of schizophrenia

Social domain nursing diagnosis for schizophrenia

Impaired social interaction Ineffective role performance Dysfunction family process Interrupted family process

As clients are leaving the dayroom following a group therapy session, the nurse notices a client admitted for acute mania is clenching and unclenching both fists, swearing, and glaring at a staff member. Which action should the nurse take first? A. Calmly ask the client to go to the "quiet room." B. Instruct clients to return to the dayroom. C. Prepare to administer a sedative medication. D. Ask a staff member to call hospital security.

Instruct clients to return to the dayroom.

A client who has been diagnosed with Bipolar I Disorder states, "God has taught me how to decode the Bible." The nurse should anticipate which combination of medications would be ordered to address this client's symptoms? A. Lithium carbonate (Lithobid) and risperidone (Risperdal) B. Lithium carbonate (Lithobid) and carbamazepine (Tegretol) C. Valproic acid (Depakote) and sertraline (Zoloft) D. Valproic acid (Depakote) and lamotrigine (Lamictal)

Lithium carbonate (Lithobid) and risperidone (Risperdal)

A client who has been diagnosed with bipolar I disorder states, "God has taught me how to decode the Bible." A nurse should anticipate that which combination of medications would be ordered to address this client's symptoms?

Lithium carbonate (Lithobid) and risperidone (Risperdal)

A client diagnosed with Bipolar Disorder, who has taken lithium carbonate (Lithane) for 1 year, presents in an emergency department with severe diarrhea, blurred vision, and tinnitus. The nurse should interpret these symptoms to be indicative of which of the following? A. Consumption of foods high in tyramine B. Lithium carbonate discontinuation syndrome C. Development of lithium carbonate tolerance D. Lithium carbonate toxicity

Lithium carbonate toxicity

Neurocognitive impairment in schizophrenia

Memory, vigilance or sustained attention, verbal fluency or the ability to generate new words, executive function(volition, planning, purposeful actions self monitoring behavior) Disorganized thinking

Catatonic disorder due to another medical condition

Metabolic disorders, neurological conditions

A patient on antipsychotic medication reports to the nurse that her muscles feel very stiff, and she appears diaphoretic. Her temperature is 105 degrees. Her symptoms are indicative of the potentially fatal adverse reaction to antipsychotic medication known as ________________________

Neuroleptic Malignant Syndrome Although neuroleptic malignant syndrome is rare, its rapid progression and potential to cause death make it a priority to assess for regularly and to intervene aggressively when symptoms are apparent. Antipsychotic medication should be immediately discontinued

Psychotic disorder associated with another medical condition

Prominent hallucinations and delusions are directly attributable to a general medical condition

A client is diagnosed with Bipolar I Disorder: Manic Episode. Which nursing intervention should be implemented to achieve the outcome of "Client will gain 2 pounds by the end of the week?" A. Provide client with high-calorie finger foods throughout the day. B. Accompany client to cafeteria to encourage adequate dietary consumption. C. Initiate total parenteral nutrition to meet dietary needs. D. Teach the importance of a varied diet to meet nutritional needs.

Provide client with high-calorie finger foods throughout the day.

A client is diagnosed with bipolar I disorder: manic episode. Which nursing intervention would be implemented to achieve the outcome of "Client will gain 2 pounds by the end of the week?"

Provide client with high-calorie finger foods throughout the day.

The nurse is planning care for a client diagnosed with Bipolar Disorder: Manic Episode. Which should be the first priority of the listed client outcomes? A. Maintains nutritional status B. Interacts appropriately with peers C. Remains free from injury D. Sleeps 6 to 8 hours a night

Remains free from injury

A nurse is planning care for a client diagnosed with bipolar disorder: manic episode. In which order should the nurse prioritize the listed client outcomes?

Remains free from injury. Maintains nutritional status. Sleeps 6 to 8 hours a night. Interacts appropriately with peers.

A client diagnosed with Bipolar Disorder: Depressive Episode intentionally overdoses on sertraline (Zoloft). Family reports that the client has experienced anorexia, insomnia, and recent job loss. Which should be the priority nursing diagnosis for this client? A. Risk for suicide R/T hopelessness B. Anxiety: severe R/T hyperactivity C. Imbalanced nutrition: less than body requirements R/T refusal to eat D. Dysfunctional grieving R/T loss of employment

Risk for suicide R/T hopelessness

A client diagnosed with bipolar disorder: depressive episode intentionally overdoses on sertraline (Zoloft). Family reports that the client has experienced anorexia, insomnia, and recent job loss. What should be the priority nursing diagnosis for this client?

Risk for suicide R/T hopelessness

A newly admitted client is experiencing a manic episode of bipolar I disorder and presents as very agitated. The nurse should assign which priority nursing diagnosis to this client?

Risk for violence: directed toward others R/T agitation and hyperactivity

An adult client diagnosed with bipolar I disorder is prescribed lamotrigine (Lamictal), 400 mg three times a day, for mood stabilization. Which is a true statement about this medication order?

This dosage is more than twice the recommended dosage range.

A newly admitted client is experiencing a manic episode of bipolar I disorder and presents as very agitated. The nurse should assign which priority nursing diagnosis to this client? A. Ineffective individual coping R/T hospitalization AEB alcohol abuse B. Altered nutrition: less than body requirements R/T mania AEB 10-pound weight loss C. Risk for violence: directed toward others R/T agitation and hyperactivity D. Sleep pattern disturbance R/T flight of ideas AEB sleeps 1 to 2 hours per night

Risk for violence: directed toward others R/T agitation and hyperactivity

The inpatient psychiatric unit is being redecorated. At a unit meeting, staff discusses bedroom décor for clients experiencing mania. The nurse manager evaluates which suggestion as most appropriate?

Rooms should be painted with neutral colors and contain pale-colored accessories.

The inpatient psychiatric unit is being redecorated. At a unit meeting, staff discusses bedroom décor for clients experiencing mania. The nurse manager evaluates which suggestion as most appropriate? A. Rooms should contain extra-large windows with views of the street. B. Rooms should contain brightly colored walls with printed drapes. C. Rooms should be painted deep colors and located close to the nurse's station. D. Rooms should be painted with neutral colors and contain pale-colored accessories.

Rooms should be painted with neutral colors and contain pale-colored accessories.

Schizophreniform disorder

Same symptoms as schizophrenia with the exception that the duration of the disorder has been at least 1 month but less than 6 months

Schizoaffective disorder

Schizophrenic symptoms accompanied by a strong element of symptomatology associated with the mood disorders, either mania or depression

Biological domain nursing diagnosis for schizophrenia

Self neglect Disturbed sleep pattern Imbalanced nutrition less than Excess fluid volume Sexual dysfunction

Brief psychotic disorder

Sudden onset of symptoms, may or may not be preceded by severe psychosocial stressor, lasts less than 1 month

Priority care issues of schizophrenia

Suicide assessment Aggression and safety of patient, staff, others Antipsychotic medication

13. The nurse in the emergency department is assessing a client suspected of being suicidal. Number the following assessment questions, beginning with the most critical and ending with the least critical. 1. _____ "Are you currently thinking about suicide?" 2. _____ "Do you have a gun in your possession?" 3. _____ "Do you have a plan to commit suicide?" 4. _____ "Do you live alone? Do you have local friends or family?"

The assessment questions should be numbered as follows: 1, 3, 2, 4.

A client is diagnosed with Cyclothymic Disorder. Which client behaviors should the nurse expect to assess? A. The client expresses "feeling blue most of the time." B. The client has endured periods of elation and dysphoria lasting for more than 2 years. C. The client fixates on hopelessness and thoughts of suicide continually. D. The client has labile moods with periods of acute mania.

The client has endured periods of elation and dysphoria lasting for more than 2 years.

A client is diagnosed with cyclothymic disorder. What client behaviors should the nurse expect to assess?

The client has endured periods of elation and dysphoria lasting for more than 2 years.

A client is diagnosed with Bipolar Disorder and admitted to an inpatient psychiatric unit. Which is the priority outcome for this client? A. The client will accomplish activities of daily living independently by discharge. B. The client will verbalize feelings during group sessions by discharge. C. The client will remain safe throughout hospitalization. D. The client will use problem-solving to cope adequately after discharge.

The client will remain safe throughout hospitalization.

A client is diagnosed with bipolar disorder and admitted to an inpatient psychiatric unit. Which is the priority outcome for this client?

The client will remain safe throughout hospitalization.

44. A client is prescribed venlafaxine (Effexor) 75 mg qam and 150 mg qhs. Venlafaxine is supplied in a 37.5-mg tablet. How many tablets would the nurse administer a day? _____ tablets.

The nurse will administer 6 tablets in 1 day.

Substance induced psychotic disorder

The presence of prominent hallucinations and delusions that are judged to be directly attributable to substance intoxication or withdrawal

The nurse begins the intake assessment of a client diagnosed with Bipolar I Disorder. The client shouts, "You can't do this to me. Do you know who I am?" Which is the priority nursing action in this situation? A. To provide self and client with a safe environment B. To redirect the client to the needed assessment information C. To provide high-calorie finger foods to meet nutritional needs D. To reorient the client to person, place, time, and situation

To provide self and client with a safe environment

A client is admitted in a manic episode of bipolar I disorder. Which nursing intervention is most therapeutic for this client? A. Use a calm, unemotional approach during client interactions. B. Focus primarily on enforcing limits. C. Limit interactions to decrease external stimuli. D. Encourage the client to establish social relationships with peers.

Use a calm, unemotional approach during client interactions.

Clozapine (clozaril)

Used when second generation antipsychotics aren't effective Decreases WBC

A client is admitted in a manic episode of bipolar I disorder. Which nursing intervention should be most therapeutic for this client?

Using a calm, unemotional approach during client interactions

A client diagnosed with Bipolar I Disorder: Manic Episode refuses to take lithium carbonate because he complains that it makes him feel sick. Which of the following medications might be alternatively prescribed for mood stabilization in bipolar disorders? A. Sertraline (Zoloft) B. Valproic acid (Depakote) C. Trazodone (Desyrel) D. Paroxetine (Paxil)

Valproic acid (Depakote)

A client diagnosed with bipolar I disorder: manic episode refuses to take lithium carbonate because he complains that it makes him feel sick. Which of the following medications might be alternatively prescribed for mood stabilization in bipolar disorders?

Valproic acid (Depakote)

A client with OCD reveals he was late to his appointment because "of my dumb habit. I have to take off my socks and put them back on 41 times! I cannot stop until I do it just right." The nurse interprets the clients behavior as MOST likely representing an effort to obtain: a. a relief from anxiety b. control of his thoughts c. attention from others d. safe expression of hostility

a

Katrina​ Wilson, a​ 20-year-old college​ student, is recently diagnosed with​ obsessive-compulsive disorder. Katrina asks you what she can do to treat her problem. Which treatment option should not be included in the answer you provide to​ Katrina? a Benzodiazepines b Cognitive behavioral therapy c Tricyclic antidepressants d Brain stimulation therapy

a (All therapies are indicated for the treatment of​ obsessive-compulsive disorder except benzodiazepines. Benzodiazepines are used for other anxiety disorders but are not useful for OCD.)

A client with​ obsessive-compulsive disorder​ (OCD) is admitted to an inpatient unit for treatment of the disorder. Which classification of medications can the nurse expect the healthcare provider to order as first line therapy for treatment of​ OCD? a Selective serotonin reuptake inhibitors​ (SSRI) b Antipsychotics c Anticonvulsants d Oral hypoglycemic agents

a (Rationale SSRIs are the​ first-line therapy for clients with OCD. Oral hypoglycemics and anticonvulsant agents are not indicated for this disorder. Antipsychotics are used in the treatment of​ OCD, but are not considered first line therapy.)

A client asks the nurse how​ cognitive-behavioral therapy will help her to manage her​ obsessive-compulsive disorder. Which response by the nurse is the most​ appropriate? a It teaches techniques that will help you lower stress. b It will make you feel shameful and therefore the behaviors will stop. c It will teach you ways to increase your​ self-esteem. d It will help you change your belief system.

a (Rationale ​Cognitive-behavioral therapy aims to teach the client adaptive coping skills that will lower the client​'s stress and anxiety. Increase in​ self-esteem may be a result of receiving the cognitive behavioral​ therapy, but it is not the goal for the client with OCD.​ Cognitive-behavioral therapy does not aim to change a client​'s belief system.​ Cognitive-behavioral therapy will assist the client with lessening feelings of shame and anxiety.)

A​ school-age male client checks and rechecks the locks on the doors of his home at least 15 times before leaving for school in the morning. The client​'s mother brings him to the healthcare provider​'s office today for a​ check-up. Which statement by the nurse is most appropriate in regard to the etiology of​ obsessive-compulsive disorder​ (OCD)? ​a Twenty-five percent of all males with OCD experience symptoms prior to age 10. b Children have a low chance of experiencing a remission of the disease. c Manifestations of OCD occur only in children. d Females are affected more than males.

a (Rationale ​Twenty-five percent of all males with OCD are diagnosed prior to the age of 10. Males and females are equally​ affected; however, the males are at an earlier age when the disease occurs. Manifestations of OCD are seen in​ children, adolescents, and young adults. Children with OCD have a​ 40% chance of experiencing remission by the time they reach adulthood.)

The nurse is caring for a child admitted with a possible diagnosis of obsessive-compulsive disorder (OCD). During the admission assessment, the nurse is gathering historical data. Which concern would the nurse ask the parents about? a History of recent streptococcus infection b History of attention deficit disorder c History of eating disorders d History of bed-wetting

a (Rationale: Studies have shown that young children experiencing OCD have recently had strep throat. It is thought that an antibody against strep acts on the brain enzymes and disrupts communication between neurons. Bed-wetting, eating disorders, and attention deficit disorder are not associated with development of OCD.)

The nurse is providing care to a client with​ obsessive-compulsive disorder​ (OCD). Which interventions are appropriate for this​ client? Select all that apply. a Encourage the client to verbalize his or her feelings. b Assist the client with developing new coping mechanisms. c Interrupt the​ ritual, using distraction. d Establish a loud and fun environment for the client. e Include time in the daily routine to perform the ritual.

a,b,e (Rationale Verbalization of feelings will assist the client with reducing stress and anxiety. The client with OCD needs to learn new coping skills to manage the intrusive thoughts that lead to the performance of rituals. Allowing time in a client​'s daily schedule to perform the ritual will allow the client to complete the ritual and still manage daily activities. A loud environment is not recommended. Interrupting a client​'s ritual may lead to increased anxiety and is not​ recommended, unless it is harmful to the client.)

The nurse is educating a client diagnosed with​ obsessive-compulsive disorder​ (OCD) on the different therapies that are available for the disorder. Which therapies are appropriate for the nurse to include in the teaching​ session? Select all that apply. a Antipsychotic medication b Antihypertensive agents ​c Cognitive-behavioral therapy d Herbal​ supplements, such as St. John​'s wort e Hypoglycemic agents

a,c (Rationale ​Cognitive-behavioral therapy is a recommended treatment for OCD to assist the client with learning new coping skills. While not a​ first-line treatment,​ antipsychotics, such as​ risperidone, may be used in the treatment of OCD when the client has not responded to selective serotonin reuptake inhibitors​ (SSRIs). No information is available on the effectiveness of herbal supplements in the treatment of OCD. Antihypertensive agents are not indicated in the treatment of OCD.)

The nurse is reviewing the health history of a client who will be seen later in the day. Which items place this client at risk for developing​ obsessive-compulsive disorder​ (OCD)? Select all that apply. a Death of the mother at a young age b A sister with depression c Sexual abuse as a child d Paternal substance abuse e Domestic abuse

a,c,e (Rationale Items in the client​'s history that increase the risk of OCD include the death of the mother at a young​ age, domestic​ abuse, and sexual abuse as a child. A sister with depression and having a father who suffers from substance abuse are not risk factors for developing OCD.)

The healthcare provider is teaching a class on obsessive-compulsive disorder (OCD). Which of the following will be included in the teaching? Choose all answers that apply: a Obsessions and compulsions are time-consuming b The patient often hears voices that direct the behavior c The behavior is sometimes related to substance abuse d The obsessions are intrusive and unwanted e Repeating words silently is an example of an obsession f Hair pulling or skin picking are common features

a,d (OCD is characterized by intrusive thoughts or urges and by repetitive mental or behavioral acts. If the patient's behaviors are related to substance abuse, the patient is not diagnosed with OCD. Patients diagnosed with OCD do not experience hallucinations. Hair pulling and skin picking are different disorders and not characteristic of OCD. OCD is characterized by intrusive thoughts or urges (obsessions) that compel the patient to perform repetitive mental or behavioral acts (compulsions). The obsessions and compulsions are typically time-consuming and often impair social, occupational, or other important areas of functioning.)

The nurse observes an adult client pacing the room and wringing his hands. The client checks the lock on the exam room door 12 times after the nurse enters the room. Which assessment questions would be appropriate when evaluating this ​client? Select all that apply. a "Does this behavior interfere with your daily ​life? ​b "What would happen if you were​ dead?" c "Does anyone in your family suffer from ​depression? " d "How old were you when you first started this ​behavior? " ​e "Are you easily​ annoyed?"

a,d (Rationale Appropriate questions for the nurse to include in the assessment of this client include asking the client when the behavior started and if it interferes with daily life. The other questions are not helpful in evaluating this client.)

The nursing instructor is preparing to educate a group of students on the risk factors of obsessive compulsive disorder (OCD). Which are risk factors for developing OCD? (Select all that apply.) a A history of childhood physical abuse b Being an adolescent male c Being an adolescent female d Having a first-degree relative with the disorder e A history of childhood sexual abuse

a,d,e (Rationale: Risk factors for developing obsessive compulsive disorder include having a first-degree relative with this disorder and a history of childhood sexual or physical abuse. It affects men and women equally; however it does develop earlier in men. The severity of obsessive-compulsive disorder is measured by the amount of time the client spends in the compulsive behavior mode; it ranges from mild, which is less than 1 hour, to extreme, which is constant. The client with 1-3 hours of behavior per day is displaying moderate OCD, and the client with 3-8 hours of behavior per day is displaying severe OCD.)

At an inpatient​ facility, the client with obsessive compulsive disorder​ (OCD) counts the number of tiles on the floor each morning before going to breakfast. This ritual takes the client 30 minutes and the client always misses breakfast. Which interventions by the nurse can assist the client in arriving to breakfast prior to the meal​ ending? Select all that apply. a Allow additional time in the client​'s morning routine to include ritual before breakfast. b Escort the client to the dining area and place in restraints. c Cancel breakfast and make the client wait until lunch to eat. d Interrupt the client​'s ritual and demand that he go to breakfast now. e Set limits on the amount of time the client performs the ritual.

a,e (Rationale: Allowing additional time for the client to complete the ritual and setting limits on the amount of time the client is allowed to complete the ritual are therapeutic interventions for this client. Interrupting the client​'s ritual causes additional stress and anxiety and is not therapeutic. Cancelling the client​'s breakfast is not therapeutic. Placing the client in restraints is inappropriate.)

The nurse is caring for a client with​ obsessive-compulsive disorder​ (OCD) . Which clinical manifestations would the nurse expect to see in this​ client? Select all that apply. a Intrusive thoughts b Physical complaints such as irritated skin c Happy and overly excited affect d Signs of distress and increased anxiety e Repetitive actions or motions

a.b.d.e (Rationale: Repetitive behaviors are a hallmark sign of OCD. Physical complaints can be seen in patients who perform repetitive activities such as hand washing. Signs of distress or increased anxiety can be seen in clients when they feel compelled to complete rituals. The ritual is the client​'s way of resolving the anxiety. A happy and overly excited affect is not a clinical manifestation associated with OCD.)

A client diagnosed with OCD has been taking sertraline but would like to have more energy every day. At his monthly chekup her reports that his massage therapist recommended he take St Johns Wort to help his depression. The nurse should tell the client: a. st johns wort is a harmless herb that might be helpful in this instance b. combining st johns wort with the sertraline can cause a serious reaction called seretonin syndrome c. if you take st johns wort we will have to decrease your sertraline. d. st johns wort is not very effective for depression but we can increase your sertraline dose

b

A client with obsessive compulsive disorder​ (OCD) washes her hands up to 100 times each day. What is this client at risk​ for? a Impaired socialization b Impaired skin integrity c Alterations in family processes d Alterations in sleep

b

A patient diagnosed with obsessive-compulsive disorder (OCD) continually carries a toothbrush, and will brush and floss up to fifty times each day. The healthcare provider understands that the patient's behavior is an attempt to accomplish which of the following? Choose 1 answer: Choose 1 answer: a Avoid interacting with others b Relieve anxiety c Promote oral health d Experience pleasure

b

Sonya​ Moore, 25 years of​ age, presents with the following​ symptoms: complaints of​ stress, fear of an intruder breaking into the​ home, and the need to constantly check and recheck all locks and doors. Sonya is diagnosed with​ obsessive-compulsive disorder​ (OCD). Which item in​ Sonya's health history placed her at an increased risk for developing​ OCD? a Paternal history of coronary artery disease​ (CAD) b Tragic car accident during childhood c Maternal history of depression d HPV immunization as an adolescent

b (One of the risk factors for developing​ obsessive-compulsive disorder​ (OCD) is a traumatic event during​ childhood, such as a tragic car accident. The HPV​ immunization, a maternal history of​ depression, and a paternal history of coronary artery disease​ (CAD) are not risk factors for coronary artery disease​ (CAD).)

The nurse educator is teaching a group of students about​ obsessive-compulsive disorder​ (OCD). Which statement will the educator include in the teaching session regarding​ OCD? a Diagnosis of OCD is easy. b Children who have had a streptococcal infection may be at risk of developing the disorder. c Brain imaging in clients with OCD is normal. d Signs and symptoms of OCD occur in older adults.

b (Rationale Children with previous streptococcal infections are at risk for developing OCD. Clinical manifestations of OCD typically occur in children and young adults. Diagnosis of OCD is difficult due to the variations in clinical presentation. Brain imaging in clients with OCD is abnormal.)

The nurse teaches a client about medications used in the treatment of​ obsessive-compulsive disorder​ (OCD). Which client statement indicates appropriate understanding of the teaching​ session? a "Medications are not effective in the treatment of OCD. " b "I may only have to take medication for​ 1-2 years and gradually be weaned " c "I will have to take medication for the rest of my life. " d "There are no side effects associated with this medication.

b (Rationale Clients with a successful medication regimen may be on therapy for 1dash-2 years and then be tapered off the medication while observing for symptom exacerbation. Individual client response will determine whether the client needs to be on medications for​ short-term or​ long-term use. All medications have side effects and clients should be taught about the side effects. Medications are effective in the management of OCD.)

Which would the nurse consider an obsession? a Repetitive behaviors b Recurrent and persistent thoughts or images c Counting d Acts aimed at reducing anxiety

b (Rationale: Obsession is defined as a recurrent or persistent, unwanted thought or image that causes marked anxiety and distress and results in actions or compulsions. Compulsion is a need to perform an action aimed at reducing anxiety such as repetitive behaviors or a mental activity such as counting)

Which pharmacologic agent is used in the treatment of​ obsessive-compulsive disorder​ (OCD)? a Sulfonlyureas b Selective serotonin reuptake inhibitors c Diuretics d ACE inhibitors

b (Selective serotonin reuptake inhibitors are used in the treatment of OCD. ACE inhibitors are not used in the treatment of OCD. Sulfonylureas are not used in the treatment of OCD. Diuretics are not used in the treatment of OCD.)

A client with OCD who was admitted early yesterday morning, must make his bed 22 times before he can have breakfast. Because of this behavior, the client missed having breakfast yesterday with the other clients. Which action should the nurse institute to help the client be on time for breakfast? a. Tell the client to make his bed only one time b. wake the client an hour earlier to perform his ritual c. insist that the client stop his activity when it is time for breakfast d. advise the client to have breakfast first before making his bed

b (the nurse should never take away a ritual, panic will ensue, the nurse should work with the client later to slowly set limits on frequency of the action)

The nurse, caring for a client who demonstrates obsessive-compulsive disorder including symmetry obsessions with ordering, would expect to assess which behaviors in this client? (Select all that apply.) a Frequently checking that doors are locked at night b Repeating movements c Brushing the teeth seven times a day d Compulsive acquisition of items on sale e Counting or spelling silently or out loud

b,e (Rationale: Symptoms of symmetry obsessions might include the need to have objects in fixed and symmetrical positions, repetitive movements, and counting or spelling. Frequent checking of door locks is a typical checking compulsion. Brushing the teeth repeatedly is common with cleaning compulsions, and acquiring unneeded items is a collecting compulsion.)

Which are nursing interventions for clients with​ obsessive-compulsive disorder? Select all that apply. a Increase demands on the client b Teach about prescribed medications c Plan client teaching after completion of rituals d Encourage alternate activities to distract from compulsions e Interrupt ritual behaviors to discourage the client from doing them

b.c.d (Nursing interventions for clients with OCD include encouraging alternate activities to distract from​ compulsions, planning client teaching after completion of​ rituals, and teaching clients about prescribed medications. You would not interrupt ritual​ behaviors, as​ clients' feelings of having control and mitigating anxiety are closely associated with completing these behaviors. You would decrease demands on the​ client, not increase them.)

A nurse observes a client who has OCD repeatedly applying removing and reapplying makeup. The nurse identifies that repetitive behavior in a client who has OCD is due to which of the following underlying reasons? a. narcissistic behavior b. fear of rejection from staff c. attempt to reduce anxiety d. adverse affect of antidepressant med

c

During a panic attack, a patient states, "I feel like I'm going to die!" The patient is hyperventilating, tachycardic, and reports feeling upper extremity numbness and tingling. Based on this patient's presentation, the healthcare provider would anticipate which additional clinical manifestation of the panic attack? Choose 1 answer: A Kussmaul respirations B Respiratory acidosis C Respiratory alkalosis D Hypercapnia

c

When caring for a patient during an acute panic attack, which of the following actions by the healthcare provider is most appropriate? Choose 1 answer: Choose 1 answer: A Ask open-ended questions to encourage communication B Use distraction techniques to change the patient's focus C Offer the patient reassurance of safety and security D Explore common phobias associated with panic attacks

c

The nurse caring for a 10-year-old who is being evaluated for a mental disorder suspects that the child has an alteration in serotonin synthesis when which disorder is observed? a Specific phobias b Agoraphobia c Childhood obsessive-compulsive disorder (OCD) d Panic attacks

c ( Feedback Rationale: There is an alteration in serotonin synthesis in the brains of children and adolescents who develop OCD. The chemical imbalance is not associated with panic attacks, specific phobias, or agoraphobia.)

A client diagnosed with​ obsessive-compulsive disorder​ (OCD) tells the nurse that he has had feelings of apprehension that are alleviated through frequent hand washing. The​ client's hands are​ red, swollen, and the nurse notes several areas of excoriation. Based on the assessment​ findings, which nursing diagnosis is a priority for this​ client? a Impaired coping mechanisms b Alterations in sleep c Impaired skin integrity d Increased risk for anxiety

c (Learning Objective The client who is exhibiting skin breakdown is experiencing the nursing diagnosis of impaired skin integrity. While the other nursing diagnoses are appropriate for a client with​ OCD, they are not a priority for this client.)

The nurse teaches the family of a child recently diagnosed with​ obsessive-compulsive disorder​ (OCD) what to expect from their child. Which statement by the family indicates effective​ client/family teaching? a "I will interrupt my son​'s counting every morning. b "My son only needs to take his medication and then his symptoms will disappear. " c "I will allow additional time for breakfast in order for my son to complete his ritual " d "My son will never get better.

c (Rationale Additional time will allow the client to complete the ritual and continue with other activities. Interrupting the client​'s ritual will only increase the client​'s stress and anxiety. A statement indicating that the son will never get better indicates the need for further education. Medication alone is often not effective in the treatment of OCD. A combination of​ cognitive-behavioral therapy and medication is recommended.)

The nurse is assessing the mental health of a female adult client who has been under stress at work. The client wants to wash her hands every 2 to 3 minutes and wipes the flat surface areas in the clinic with a paper towel while talking to the nurse. Which aspect of​ obsessive-compulsive disorder​ (OCD) is this client​ exhibiting? a Auditory hallucinations b Visual hallucinations c Repetitive behavior d Illogical thinking

c (Rationale Clients diagnosed with OCD repeat behaviors over a short period of time in the absence of the need to perform these behaviors. Visual and auditory hallucinations are seeing things that are not there and hearing things that are not​ there, respectively, not repeated behaviors. Illogical thinking involves thoughts and ideas running together in the thought​ process, not repeated behaviors.)

A client with​ obsessive-compulsive disorder​ (OCD) tells the nurses that he checks the locks to his house 10 times before leaving in the morning for work. The​ client's mother does not understand why her son does this. The nurse bases her response on which rationale for the​ behavior? a Call attention to himself b Check the safety of the home c Reduce anxiety d Control others

c (Rationale The only statement that is correct in this scenario is reducing anxiety. Clients with OCD perform rituals to control intrusive​ thoughts, not to check the safety of the​ home, to control​ others, or to call attention to themselves.)

Which medication would the nurse expect to administer to a client who is experiencing ritualistic behavior that interferes with job performance and activities of daily living? a Fluphenazine (Prolixin) b Lorazepam (Ativan) c Fluoxetine (Prozac) d Carbemazepine (Tegretol)

c (Rationale: Fluoxetine is a selective serotonin reuptake inhibitor (SSRI) that corrects the imbalance of serotonin in anxiety disorders such as obsessive-compulsive disorder. Fluphenazine is an antipsychotic and not appropriate for anxiety disorders. Lorazepam is a sedative with central nervous system depressive side effects that is used mostly for sedation and to manage status epilepticus. Tegretol is used mainly for seizure disorders.)

A client with obsessive-compulsive disorder (OCD) is continuously washing the hands and bathing. The nurse concludes that this client has a fear of contamination. Which diagnosis is priority for this client? a Deficient Knowledge b Stress Overload c Fear d Anxiety

c (Rationale: The client is exhibiting fear of contamination and exposure to germs; therefore the priority diagnosis would be fear. The client is not demonstrating a knowledge deficit. While the client may also be experiencing stress and anxiety they would not be the priority diagnosis.)

A client diagnosed with​ obsessive-compulsive disorder​ (OCD) is being discharged home. Which teaching point is appropriate to​ include? a Drink wine in the evenings to promote sleep b Eat meals high in fat content c Avoid caffeinated beverages d Take medication only when needed

c (The nurse would teach the client to avoid caffeinated beverages. Medication should be taken as prescribed. Alcohol should be limited or avoided. The client should eat a healthy​ diet, not one that is high in fat content.)

Jennifer​ Shubert, a​ 20-year-old college​ student, is seeking treatment for​ obsessive-compulsive disorder​ (OCD). You enter the examination room and prepare to complete the psychosocial history for this client. Which response to the client is most appropriate for the psychosocial history portion of the nursing​ assessment? ​a "Will you roll up your sleeve so I can take your blood​ pressure?" ​b "I must complete a head to toe examination. Can you change into this​ gown?" ​c "I see that you have a history of substance abuse. What was your drug of​ choice?" ​d "Would it be ok if I looked at your​ skin? Your hands appear to be​ excoriated."

c (While assessing the​ client's psychosocial​ history, it is appropriate for the nurse to determine the substance abuse​ history, including the drug of choice. All the other responses by the nurse are more appropriate during the physical examination portion of the nursing assessment.)

The nurse is conducting a nursing assessment for a client diagnosed with​ obsessive-compulsive disorder​ (OCD). Which findings are indicative of the repetitive acts associated with ​OCD? Select all that apply. a Underweight and appears older than stated age b Poor posture and altered motor skills c Constant hand washing d The need to lock and unlock doors e Poor grooming and stained clothing

c,d (Rationale Assessment findings that illustrate the repetitive acts associated with OCD include constant hand washing and the need to lock and unlock doors. The other findings would not support the repetitive acts associated with OCD.)

Which comment made by a client diagnosed with obsessive compulsive disorder (OCD) would indicate to the nurse a positive outcome? a "I don't see what is wrong with washing my hands every hour." b "If I cannot spell everything that I read aloud, I would rather stay in bed all day." c "I no longer wash my hands every 5 minutes but I do clean the door knobs every hour." d "I am able to sleep through the night and no longer get up every hour to check that the door is locked."

d ( Feedback Rationale: This comment demonstrates a reduction in the performance of a ritualistic behavior. The other comments demonstrate denial and inadequate coping skills.)

Which symptom pattern is NOT indicative of a client with​ obsessive-compulsive disorder? a Hoarding b Nail biting c Trichotillomania d Randomly placed objects

d (A client with​ obsessive-compulsive disorder has a need for symmetry and precision and therefore would not tolerate randomly placed objects. Trichotillomania​ (hair pulling),​ hoarding, and nail biting are all symptom patterns commonly exhibited by clients with​ obsessive-compulsive disorder.)

Which nursing diagnosis is appropriate for a client diagnosed with​ obsessive-compulsive disorder​ (OCD)? a Potential for infection b Impaired airway clearance c Increased activity tolerance d Alterations in sleep

d (An appropriate nursing diagnosis for a client diagnosed with OCD is alterations in sleep. Potential for​ infection, increased activity​ tolerance, and impaired airway clearance are not appropriate nursing diagnoses for this client.)

Which of the following defense mechanisms may be observed in a patient diagnosed with obsessive-compulsive disorder (OCD)? Choose 1 answer: Choose 1 answer: a Regression b Projection c Denial d Undoing

d (Defense mechanisms are used when there is a threat to the patient's psychological integrity. The patient diagnosed with OCD will use this defense mechanism to deal with intolerable levels of anxiety. The patient attempts to substitute the anxiety with a behavior which is maladaptive. Undoing is a way of symbolically canceling out (negating) an experience that the patient finds intolerable. The compulsive behavior is designed to counteract or undo the patient's obsession.)

How is​ obsessive-compulsive disorder​ (OCD) diagnosed? ​a Cognitive-behavioral therapy b Psychotherapy c Deep brain stimulation d Client history

d (OCD is often diagnosed through the client history.​ Psychotherapy, cognitive-behavioral​ therapy, and deep brain stimulation are treatment options for OCD.)

A​ 21-year-old female​ client, Grace, presents to the urgent care center with complaints of​ irritated, dry, and bleeding skin on her hands. As the nurse is talking to the​ client, you observe her get up and go to the sink and wash her hands five times. Which nursing intervention is not appropriate for​ Grace? a Refer the client for cognitive behavioral therapy b Teach the client alternative coping mechanisms c Administer SSRIs as ordered d Interrupt the​ client's hand washing ritual and tell her it is not necessary for her to wash her hands five times

d (Referral for​ cognitive-behavioral therapy, administering an SSRI as​ ordered, and teaching the client coping mechanisms are all appropriate nursing interventions. Interrupting the client may increase the​ client's anxiety and telling the client that it is not necessary to wash her hands so often may increase the​ client's feelings of shame.)

Age of onset of schizophrenia

late adolescence and early adulthood


Conjuntos de estudio relacionados

Chapter 38: Caring for Clients With Cerebrovascular Disorders

View Set

AP Euro Ch. 20: "The Revolution in Politics"

View Set

Plants: Unit 7 Quiz 3 Study Guide

View Set

PrepU Chapter 66 Neurological Disorders

View Set

Digestive System, Section 1: Exercise 5

View Set

RHIT Practice Exam 1 - 150 Questions

View Set

Common Logical Fallacies: Examples

View Set

Infection Review PREP U ( NUR2 TEST 2)

View Set